由买买提看人间百态

boards

本页内容为未名空间相应帖子的节选和存档,一周内的贴子最多显示50字,超过一周显示500字 访问原贴
Medicalpractice版 - [合集] 你有生以来做出的最佳临床诊断是什么? (转载)
相关主题
An interesting case to share大家说说, 以你自己的理解, 医学是什么东东?
【Case discussion】 Vision lossHeparin drip
Don't jump, please新医生上路最常犯的9个错误
病例有奖竞猜, Ageusia and anosmiaFunny story about OSH
Case #2 Fever in Traveler有包子!求macrocytic anemia 的鉴别诊断
肥胖是一种疾病逆风妹妹的小说哪去啦?
【病例讨论】Anisocoria哈医血案
【有奖征贴】说一说你亲眼见过的最疑难病例求教:妈妈冠心病的治疗选择(附CT图)
相关话题的讨论汇总
话题: ct话题: els话题: pain话题: pe话题: mestinon
进入Medicalpractice版参与讨论
1 (共1页)
s********o
发帖数: 3319
1
【 以下文字转载自 MedicalCareer 讨论区 】
发信人: herby (小臭宝), 信区: MedicalCareer
标 题: [合集] 你有生以来做出的最佳临床诊断是什么?
发信站: BBS 未名空间站 (Thu Jun 2 00:01:26 2011, 美东)
☆─────────────────────────────────────☆
Aplusplus (Hakuna Matata) 于 (Sun Mar 27 11:11:49 2011, 美东) 提到:
首先我申明,我放弃我的ignore list, 欢迎所有ID参加。
每个医生护士或者其他相关人员, 无论在中国还是美国, 肯定都有一些case刻骨铭心
。 请大家把你印象最深的一例写出来, 与大家分享。 比方说, 清华男被鉈杀案,
那个中国来的护士就是诊断的关键, 相信本例就是她一辈子也忘不了的诊断。
我觉得这样的讨论有助于大家提高学医热情,丰富医学知识,开拓临床思维, 间接帮
助考版医生复习,准备CS,和 interview. 你要是能从中挑一个case让你PD
impressed,我想我的目的就达到了。
希望所有ID, 包括考生, resident, fellow, attending,PA/NP, nurse 和中国来
的医生不再从事医疗工作的, 都能奉献你的pearl. 英文中文都行。
我先来:
CC: Left leg pain for 5 days.
35 yrs old female woke up with left foot pain. Saw her PCP, had x-rays, and was
given motrin. No improvement but worse with left leg pain as well. See
another PCP again, diagnosis was sciatica. She was given neurontin. No
effect. She came to me on herself. (walk-in)
After my exam, I agreed with her PCP's assessment and management. But before
the patient put on her socks, I noticed the left foot skin color was a
little different from the other side. So I put my hands on both her feet
simutanously: One was warm and the other was "ice" cold.
I ordered a stat vascular U/S.
The patient had an emergent surgery for left leg popliteal artery thrombosis
. She was a heavy smoker.
☆─────────────────────────────────────☆
Aplusplus (Hakuna Matata) 于 (Sun Mar 27 11:17:33 2011, 美东) 提到:
I forgot to attach the picture.
☆─────────────────────────────────────☆
abcd2010 (abc) 于 (Sun Mar 27 11:59:14 2011, 美东) 提到:
您应该也整个专线。
☆─────────────────────────────────────☆
herby (迎春) 于 (Sun Mar 27 12:02:42 2011, 美东) 提到:
Buerger’s Disease?
☆─────────────────────────────────────☆
Aplusplus (Hakuna Matata) 于 (Sun Mar 27 12:19:35 2011, 美东) 提到:
Right,Buerger’s Disease is vasculitis and its symptoms could be close to this case.
I believe she has ischemic PVD due to smoking.
希望case 不单单是最后诊断。
You can present the case as a brief SOAP Note:
1. Subjectives, 病人的症状
2. Objectives, 你的发现, 包括PE, lab, images, etc
3. Assessment, 你的诊断, 为什么。
4. Plan。 治疗意见, 有好的结果最佳!
☆─────────────────────────────────────☆
Aplusplus (Hakuna Matata) 于 (Sun Mar 27 12:31:13 2011, 美东) 提到:
你也来个case吧,不需要很复杂的。
我强调的是过程, 不光是结果。
大家随便说说, 我已经抛砖了。。。
☆─────────────────────────────────────☆
chugol (看风景) 于 (Sun Mar 27 13:34:22 2011, 美东) 提到:
你觉得为什么之前的PCP误诊的原因是什么?
经验不够?还是这个病人的病程决定的?譬如说在这个病人在看你之前她的症状还没有
这么坏,artery thrombosis in its early stage, for instance.
你个人从这个病例中总结出的经验是什么呢?
☆─────────────────────────────────────☆
Aplusplus (Hakuna Matata) 于 (Sun Mar 27 14:00:53 2011, 美东) 提到:
Q: 你觉得为什么之前的PCP误诊的原因是什么?
A: 没有A++的水准? LOL
Q: 经验不够?还是这个病人的病程决定的?
A: 可能两者皆有。 你其实回答了你自己的问题。
Q:你个人从这个病例中总结出的经验是什么呢?
A: 1. 不要相信Telemedicine. 有诱惑, 曾经想在家工作,后来觉得risk太大。
2. 细节一定不要放过, 我想我的诊断来自观察, 愿意蹲下摸她的“臭脚”,LOL
3. 不要想当然take existing diagnosis.
☆─────────────────────────────────────☆
rainingcats (喵喵) 于 (Sun Mar 27 15:10:51 2011, 美东) 提到:
赞!
不过我没什么经验,围观一下
this case.
☆─────────────────────────────────────☆
cqfly (cq) 于 (Sun Mar 27 15:34:38 2011, 美东) 提到:
同赞,排队围观.
☆─────────────────────────────────────☆
chestnuts (chestnut) 于 (Sun Mar 27 15:43:33 2011, 美东) 提到:
year ago.
did not see the patient. History by pt's daughter, my co-worker
53 years old femal, no medical history, c/o both leg weakness. Pt wen to a
clinic getting some Chinese meds IV drip, no improvment. LOL
no PE
postive family history: brother and sister hypothyrodism
suggestion: CMP and thyroid hormone
Feedback: she was diagnosed as hyperthroidsm
no detail, as I said, not my patient.
I was so proud, because this co-worker treated me totally different after
that.
☆─────────────────────────────────────☆
stardust (水晶) 于 (Sun Mar 27 17:55:28 2011, 美东) 提到:
Firstly, thanks LZ for the great thread. I will follow once I have a good
one.
@chestnuts: You are great! did you make the suggestion by their history or
something else?
☆─────────────────────────────────────☆
abcd2010 (abc) 于 (Sun Mar 27 18:19:04 2011, 美东) 提到:
我有生以来给人最伟大的诊断就是: Alzheimer's disease, 但好像被版主给删了。不支持我的诊断。
☆─────────────────────────────────────☆
Aplusplus (Hakuna Matata) 于 (Sun Mar 27 19:17:05 2011, 美东) 提到:
Interesting.
Thyroid disease presentations are so non-specific that the diagnosis is
somewhat difficult. I remember when I was an medical student in China, one
of my classmates made a diagnosis of Grave's disease at his cardiology rotation
after the patient was misdiagnosed as heart disease by multiple senior
residents and attendings. He ended up being an endocrinologist.
☆─────────────────────────────────────☆
chestnuts (chestnut) 于 (Sun Mar 27 19:23:27 2011, 美东) 提到:
Hyperthyroidism can cause myositis, periodic paralysis, low potassium, etc,
either can lead to weakness.
So far I am still not clear the family trend of thyroid disease, but
periodic paralysis, or periodic hypokelamia has family trait.
Got thoes from my Endo rotation, I had a patient who had myostitis secondary
to hyperthyroidism.
☆─────────────────────────────────────☆
SUMO2009 ((MAP)) 于 (Sun Mar 27 19:29:55 2011, 美东) 提到:
One of the students told me exactly the same story when he did his
rotation in our department.
one
rotation
☆─────────────────────────────────────☆
Aplusplus (Hakuna Matata) 于 (Sun Mar 27 20:30:02 2011, 美东) 提到:
Smart people thinks same, lol.
P.S. Dr.Newbie says: Great minds think alike.
☆─────────────────────────────────────☆
Aplusplus (Hakuna Matata) 于 (Sun Mar 27 20:39:51 2011, 美东) 提到:
Alzheimer's diagnosis = great diagnosis. Are you sure?
I know Ronald Reagan started to have symptoms 13 years prior to the
diagnosis. If you were able to call him "an idiot" during his presidency,
which I bet he had been called multiple times, you were probably great. But
that kind of call does not take a doctor's degree to make, lol.
Bring it up, let us enjoy...

不支持我的诊断。
☆─────────────────────────────────────☆
Aplusplus (Hakuna Matata) 于 (Sun Mar 27 20:48:49 2011, 美东) 提到:
That's why we always check TSH for chronic fatigue syndrome.
,
secondary
☆─────────────────────────────────────☆
Aplusplus (Hakuna Matata) 于 (Sun Mar 27 21:01:08 2011, 美东) 提到:
We are waiting...
I know everyone has at least one great case, how about our BZ/BFs?
☆─────────────────────────────────────☆
skyscorpio (天之蝎子) 于 (Sun Mar 27 22:28:04 2011, 美东) 提到:
赞一个!
贵在坚持哈 :)
☆─────────────────────────────────────☆
Aplusplus (Hakuna Matata) 于 (Sun Mar 27 22:32:36 2011, 美东) 提到:
你来一个case?
☆─────────────────────────────────────☆
skyscorpio (天之蝎子) 于 (Sun Mar 27 22:38:00 2011, 美东) 提到:
掩面疾走
☆─────────────────────────────────────☆
Aplusplus (Hakuna Matata) 于 (Sun Mar 27 23:39:15 2011, 美东) 提到:
C'mon, don't go away.
Ok, since you do not want to contribute, please read this:
When your program director interviews you and asks you:“I noticed on your
CV your nickname at mitbbs is Scorpion, would you mind telling me why
chinese people eats Scorpion? "(see picture)
Your answer?
☆─────────────────────────────────────☆
abcd2010 (abc) 于 (Mon Mar 28 00:24:24 2011, 美东) 提到:
照片上的吃客不一定是Chinese。
☆─────────────────────────────────────☆
skyscorpio (天之蝎子) 于 (Mon Mar 28 10:21:23 2011, 美东) 提到:
uPig?
LOL
your
☆─────────────────────────────────────☆
Aplusplus (Hakuna Matata) 于 (Mon Mar 28 12:28:33 2011, 美东) 提到:
Actually the answer is for pain.
Where is Dr.Lexian?
iPig
☆─────────────────────────────────────☆
Lexian (蒙古大夫) 于 (Mon Mar 28 15:10:16 2011, 美东) 提到:
oh yeah? u sure? i thought ppl eat it because it tastes like shrimp. lol
scorpion poison might have some anesthetic effect, but not a bbq one like in
your picture though. hehe
☆─────────────────────────────────────☆
pineyan3 (pineyan3) 于 (Mon Mar 28 15:55:35 2011, 美东) 提到:
I have a sad case.
I was rotating at a US #1/2 cancer center.
A 40 Yr old man came in due to HA. He was diagnosed with Burket's lymphoma
and had chemotherapy several weeks ago. his labs shows leukopenia that is
likely from the chemo. He had CT, MRI that are showed white matter changes
and some sinus wall thickening. I did an LP which showed several lymphocytes
. I asked the neurosurgery to perform a biopsy but they refused. My top 1
differential was Mucor. since i was only a starting second year resident, no
one wants to listen to me, including my attending (who was a good
oncologist), ID atttending. They all think it is lymphoma involvement of the
brain. anyways, the patient lost his left eye sight on the second day of
admission, his right eye sight on thirdday, then paralysed on one side the
fourth day. he died in 1 week. an autopsy showed mucomycosis.
it is one of the reasons that I did not choose oncology as my subspecialty.
I just can't forget the many patients who died during my two months of
oncology rotation. it just makes me sad.
☆─────────────────────────────────────☆
chestnuts (chestnut) 于 (Mon Mar 28 16:12:01 2011, 美东) 提到:
interesting, lyphoma, chemo, lyphoma involvment vs mucor infection, sinus
wall thickening.
just wonder is there other way to test the spinal fluid for mucor infection
, or biopsy is the only way.
lymphocytes
no
the
☆─────────────────────────────────────☆
Aplusplus (Hakuna Matata) 于 (Mon Mar 28 19:40:53 2011, 美东) 提到:
Great case. Pearls in this case include:
1. Immunosuppressed patients are vulnerable to fungus infection. When
headache is present, CNS infection should be considered, besides the
leptomeningeal carcinomatosis.
2. CSF neg does not necessorily R/O CNS infection.
3. CNS Mucor most likely comes from invasion of sinus mucor infection
directly. If that was the case, you might consult ENT instead of NSG. If
sinus mucor can be comfirmed, treat the sinusitis aggressively instead of
encephalitis.
4. Avoid brain biopsy as much as possible if you suspect an infection.
Do you remember how many cells(mono, not lymph in CSF)exactly? What was
protein/glucose? What was india ink? what about fungal culture?
At any rate, given the CNS fungal infection, chance to survive is almost nil
.
lymphocytes
no
the
☆─────────────────────────────────────☆
Aplusplus (Hakuna Matata) 于 (Mon Mar 28 19:53:07 2011, 美东) 提到:
I believe you can do Mucor PCR.
infection
☆─────────────────────────────────────☆
Aplusplus (Hakuna Matata) 于 (Mon Mar 28 20:13:57 2011, 美东) 提到:
I think Scorpion may be good for Rheumatoid Arthritis(RA) which causes
chronic pain. So it is deemed as analgesics.
If Scorpion has ture analgesic effect, the patient may just have a breakfast
of scorpion prior to surgery. No more needs for Dr.Lexian. And you just
need to set up a farm for raising scorpion instead. lol.
I don't think anyone eats scorpion. Just for medical use. Someone exaggerates it.
in
☆─────────────────────────────────────☆
NPtobe (tiantian) 于 (Tue Mar 29 02:29:37 2011, 美东) 提到:
Also make sure check B12 level on old people who coming in with MS changes
or psychosis.
☆─────────────────────────────────────☆
NPtobe (tiantian) 于 (Tue Mar 29 02:30:22 2011, 美东) 提到:
Also make sure check B12 level on old people who coming in with MS changes
or psychosis.
☆─────────────────────────────────────☆
NPtobe (tiantian) 于 (Tue Mar 29 02:49:52 2011, 美东) 提到:
38yr, female, present with diarrhea for 4 days, initially loose watery
diarrhea, 3-4 times/days, now every hour, and blood mixed with the stool,
medium volume, non-foul smelling, with mild crampy abd pain. Had a course of
antibiotics (Keflex) 2 weeks ago
Labs:
LFTs/Amylase/Lipase: WNL
Chemo: Na 137 K 3.4 Cl 108 CO2 23 BUN 6 Cr 0.6 Glu
127
CBC: WBC 13.6 Hgb 12.4 HCT 37.9 Plt 264
Coag: PT 12.2 INR 1.1 PTT 29
Hospital Course:
1. IVF, po hydration
2. Stool Cultures, Stool O&P *3, Stool C.diff *3
3. Abx: Flagyl 500mg po Q8hrs + Levaquin 750mg IV qd
4. CT A/P with contrast was performed, which showed sigmoid colon mild to
moderate wall thickening, possibly representing colitis of infectious
etiology, with inflammatory bowel disease not excluded.
5. GI consulted, for Flex Sigmoidoscopy
☆─────────────────────────────────────☆
Aplusplus (Hakuna Matata) 于 (Tue Mar 29 17:42:28 2011, 美东) 提到:
Would you please finish your case? Please also tell us why this case is so
imressive to you.
Thanks.
of
☆─────────────────────────────────────☆
Mindset (Tough) 于 (Wed Mar 30 00:18:55 2011, 美东) 提到:
House is one of my favorite shows. In one episode, House started one treatment
plan cause all other possibilities are fatal. Was it why other attendings did
take mucor seriously?
My best diagnosis: Lupus.
☆─────────────────────────────────────☆
Aplusplus (Hakuna Matata) 于 (Wed Mar 30 12:49:11 2011, 美东) 提到:
Would you present the case of lupus? I can not wait.....
treatment
attendings did
☆─────────────────────────────────────☆
Aplusplus (Hakuna Matata) 于 (Fri Apr 1 21:26:42 2011, 美东) 提到:
Weekend case:
52 yrs old male, single, lives along, was sent to ER by the girlfriend for
new onset confusion. He was initially hallucinating and become drowsy and
progressively worse over 2-3 days. He just saw his neurologist a day before
and was sent to home, but no details available. He was directly admitted to
ICU.
PMH: ETOH abuse with pancreatitis. But he quits ETOH for 15 years. He has been seeing his neurologist for chronic pancreatits per his girlfriend.
Social: No current smoking or ETOH abuse. Used cocaine 1 month ago, but this
time all drug screen is neg.
PE: T: 100.4 F, BP 140/90, HR: 100, Stuporous,combative, sweating,
periodic myoclonus of all extremities. No focal weakness. Neg Babinski.
CT of Brain is neg. CBC, CMP and NH3 is normal. Drug screen is neg.
LP is neg.
He got Ativan 6 mg total and Haldol 5 mg, no changes.
A dose of Drug A dramatically reversed his symptoms.
BZ for first right answer. BZ/BFs, how much are you going to sponsor? I'll
match...
☆─────────────────────────────────────☆
skyscorpio (天之蝎子) 于 (Fri Apr 1 22:19:53 2011, 美东) 提到:
您老是前辈,您说奖励多少包子我们版务就奖励多少,呵呵
看着不像很难的样子,虽然我自己不知道答案 :)
before
to
this
☆─────────────────────────────────────☆
nanajj (Bumblebee Doc) 于 (Fri Apr 1 22:20:58 2011, 美东) 提到:
Serotonin syndrome - cyproheptadine
before
to
this
☆─────────────────────────────────────☆
Aplusplus (Hakuna Matata) 于 (Fri Apr 1 22:24:46 2011, 美东) 提到:
Haha, 10 wb if within 10 answers.
Why don't you try it? I give u 50 if u hit it.
☆─────────────────────────────────────☆
Aplusplus (Hakuna Matata) 于 (Fri Apr 1 22:27:30 2011, 美东) 提到:
Good thought, but there is no history supporting serotonin syndrome.
This is a real case I managed in ICU.
I just added an additional clue: I called his girlfriend and she claimed he had been seeing his neurologist for chronic pancreatitis, believe or not.

☆─────────────────────────────────────☆
chipmunk (花栗鼠) 于 (Fri Apr 1 22:47:05 2011, 美东) 提到:
From hx and available w/u, delirium tremens, intoxications, infx dz like
encephalitis, meningitis are less likely. Pt saw neurologist and might start
a neuroleptic medication. Therefore, NMS - dopamine agonist?
Thank u!
he had been seeing his neurologist for chronic pancreatitis, believe or not.
☆─────────────────────────────────────☆
Aplusplus (Hakuna Matata) 于 (Fri Apr 1 22:55:08 2011, 美东) 提到:
Sorry, he was not on any neuroleptics.
You may use D2 agonist for NMS, but more useful drug is benzo.
start
not.
☆─────────────────────────────────────☆
lll00000 (00000) 于 (Fri Apr 1 22:59:09 2011, 美东) 提到:
spontaneous bacterial peritonitis?
for
☆─────────────────────────────────────☆
Aplusplus (Hakuna Matata) 于 (Fri Apr 1 23:01:22 2011, 美东) 提到:
May I ask you why? There was no sign of sepsis.
☆─────────────────────────────────────☆
lll00000 (00000) 于 (Fri Apr 1 23:19:57 2011, 美东) 提到:
oops,no PE,no sepsis.
alcohol withdrawal? drug A is benzodiazepine?
☆─────────────────────────────────────☆
Aplusplus (Hakuna Matata) 于 (Fri Apr 1 23:23:51 2011, 美东) 提到:
Nope.
☆─────────────────────────────────────☆
skyscorpio (天之蝎子) 于 (Fri Apr 1 23:25:17 2011, 美东) 提到:
慢性胰腺炎去看神经科,有意思
☆─────────────────────────────────────☆
Aplusplus (Hakuna Matata) 于 (Fri Apr 1 23:32:28 2011, 美东) 提到:
haha, you almost get it!
☆─────────────────────────────────────☆
Jolie2010 (dreamseeker) 于 (Fri Apr 1 23:49:49 2011, 美东) 提到:
what the blood sugar level?
☆─────────────────────────────────────☆
herby (迎春) 于 (Sat Apr 2 00:10:43 2011, 美东) 提到:
偶去google了一下,瞎说几句: 慢性胰腺炎的神经痛?然后用了opioid,然后出现了
withdrawal syndrome?
☆─────────────────────────────────────☆
carrie66 (麦地听风) 于 (Sat Apr 2 00:16:50 2011, 美东) 提到:
Morphine to control Chronic Pancreatitis Neuropathic Pain--Naloxone
for
and
before
admitted to
has been seeing his neurologist for chronic pancreatits per his
girlfriend.
but this
☆─────────────────────────────────────☆
Aplusplus (Hakuna Matata) 于 (Sat Apr 2 08:29:52 2011, 美东) 提到:
Bingo!
This guy has been seeing his neurologist for chronic pain. He got his
narcotics prescription that day but did not get refill right away. Then he
developped withdrawal syndrome and became confused.
A dose of Morphine reversed his symptoms in 5 min. I started him on oral
opioids and he was discharged in 2 days.
I have to go now and great job for Herby.
☆─────────────────────────────────────☆
Aplusplus (Hakuna Matata) 于 (Sat Apr 2 08:32:55 2011, 美东) 提到:
U are very close.....
☆─────────────────────────────────────☆
herby (迎春) 于 (Sat Apr 2 10:59:29 2011, 美东) 提到:
多谢前辈,很有意思的一个case :)
he
☆─────────────────────────────────────☆
skyscorpio (天之蝎子) 于 (Sat Apr 2 11:51:17 2011, 美东) 提到:
这个例子不错!结论就是要珍惜生命,远离烟酒 :)
我转了100伪币请您代发吧
he
☆─────────────────────────────────────☆
Lexian (蒙古大夫) 于 (Sat Apr 2 14:34:40 2011, 美东) 提到:
good case. how u suggest to manage him after this fiasco?
☆─────────────────────────────────────☆
Aplusplus (Hakuna Matata) 于 (Sun Apr 3 00:05:54 2011, 美东) 提到:
Thanks.
I think the key for this case is getting a good history. If you get a good
history, you make a good diagnosis.
I posted this case 2 yrs ago in a physician web site. Nobody got right
answer: Call the girlfriend for more history.
Well, Kudos to Herby BF. Good job. That is why I always think, CMGs can do
as well as AMGs, if not even better.
蝎子BZ was right in the front of door, but he seemed to be too shy to knock
it, didn't he? lol.
I'll use up this 100 wb, many thanks to BZ.
☆─────────────────────────────────────☆
Aplusplus (Hakuna Matata) 于 (Sun Apr 3 00:06:56 2011, 美东) 提到:
I sent him to Dr.Lexian for pain management.
☆─────────────────────────────────────☆
NPtobe (tiantian) 于 (Sun Apr 3 00:24:42 2011, 美东) 提到:
38yr, female, present to PAH on 4/4/10 with diarrhea for 4 days, initially
loose watery diarrhea, 3-4 times/days, now worse and loose watery stool
every hour, and blood mixed with the stool, medium volume, non-foul smelling
, with mild crampy abd pain. Have had 5 BMs since yesterday, and the last
one almost all blood. (+) subjective fever, no chills, no cough/SOB. Had a
course of antibiotics (Keflex) 2 weeks ago (3/9/10- 3/14/10).
PMHx: Kidney stones, Seizure disorders, Vit D deficiency
PSHx: Sinus Surgery 3/10/10 for deviated nasal septum
FHx: NC
SHx: (-) smoking/ ETOH/IVDA, mental health worker
Allergies: Codeine
ROS: all negative, except as noted in HPI
PE:
Appearance: NAD
HEENT: no sclera icterus, PERRLA
PULM: B/L lungs CTAs,
CV: (+) S1, (+)S2, RRR, (-) murmurs
GI: Abd soft, mild tenderness, nondistended, (+) BS in 4 quadrant
NEURO: AAO*3
Labs:
LFTs/Amylase/Lipase: WNL
Chemo: Na 137 K 3.4 Cl 108 CO2 23 BUN 6 Cr 0.6 Glu
127
CBC: WBC 13.6 Hgb 12.4 HCT 37.9 Plt 264
Coag: PT 12.2 INR 1.1 PTT 29
Hospital Course:
Pt was admitted on 4/4/10
1. IVF, po hydration
2. Stool Cultures, Stool O&P *3, Stool C.diff *3
3. Abx: Flagyl 500mg po Q8hrs + Levaquin 750mg IV qd
4. CT A/P with contrast was performed, which showed sigmoid colon mild to
moderate wall thickening, possibly representing colitis of infectious
etiology, with inflammatory bowel disease not excluded.
5. GI consulted, for Flex Sigmoidoscopy on 4/5/10
4/5/10 AM:
Stool cultures: (-) Salmonella/Shigella/Campylobacter/Giardia/Crytosporidium
Antigen isolated in 24 hours
Flex Sigmoidoscopy: Diffuse moderate inflammation characterized by erosion,
erythema, friability, and shallow ulcerations was found in the sigmoid colon
and in the descending colon with rectal sparing, likely secondary to Crohn
’s disease. Biospies was taken with cold forceps for histology.
Dx: Crohn’s disease
Recommendations: Prednisone 40mg po QD
Outpt GI F/U in 10-14 days
4/6/10:
C.diff (-) *1
Tolerated clear liquid diet, started on low residue diet
Biopsy: mild acute colitis with crypt abscesses, no granulomata are seen, no
chronic architectural changes identified
Pt D/C’ed on prednisone 40mg po QD, F/U with GI as outpt in 10-14 days,
will consider immunomodulator or biologic agent as outpt
4/7/10:
Lab called stool culture came back: (+) shigella
Called patient at home, told her to taper steroid, and start cipro 500mg po
bid*5 days, pt is thrilled.
Requestion the patient about sick contact:
Pt is Jewish, and during Passover, had dinner with family members from upper
New York, some of them was sick.
Lesson:
Always make sure for negative stool studies before start steroid.
☆─────────────────────────────────────☆
Aplusplus (Hakuna Matata) 于 (Sun Apr 3 00:51:25 2011, 美东) 提到:
Good case.
I think the management is reasonable.
First, based on initial neg culture and positive biopsy, steroids is the
right treatment.
3 days later, shigella became positive, I wonder why not just stop steroids?
I believe in this case a tapering dose of steroids is still the right
treatment. Just like we commonly use medrol dosepack.
Again, the diagnosis would be simple if the intern got more history from the
family, LOL.
smelling
a
☆─────────────────────────────────────☆
chestnuts (chestnut) 于 (Sun Apr 3 10:26:03 2011, 美东) 提到:
I would like to present another case that is not diagnosed by me.
I took care of this 17 years old girl 2 days ago, who came to surgical floor
for VP shunt replacement for her pseudo cerebri disease. Primarily she
presented to neurologist for her 7 years history of headache, which is
constant, both sides, 4-5/10 in intensity, vague trigger factor, acerbating
two times every month,lasting one week, at the point, pain would reach 8-9/
10 in intensity.
FH: her older sister has psychiatry diagnosis. Parents are healthy, some
secondary realtives postive with autoimmunive disease.
PE: develop well, slightly malnourished .negative for other signs.
lab: Brain ct,MRI, EEG negative. Anti ssa ssb positive (for which she is
following the rheumatologist)
LP tap three times , open pressure as 20, 24. 40
pseudo cerebri is diagnosed , pt strongly requests VP shunt surgery,
immediately after surgery , the headache is gone.
The patient has been seeking for the dx and treatment for sever years, she
was diagnosed as migraine, seizure, etc.Finally they came to this
neurologist who specializes in headache.
I myself had bad headache yesterday for the whole day, I am sincerely glad
for this girl.
☆─────────────────────────────────────☆
Aplusplus (Hakuna Matata) 于 (Sun Apr 3 12:00:57 2011, 美东) 提到:
Is this girl obese?
Young female, over weight, with chronic daily headache, should be considered as
pseudotumor cerebri. On exam, you may find papillary edema. Treatment
includes diamox and LP. VP shunt is the final step. It is a brain surgery.
floor
acerbating
☆─────────────────────────────────────☆
chestnuts (chestnut) 于 (Sun Apr 3 18:06:14 2011, 美东) 提到:
She is very very slim, onset age is 10 years old. Not quite typical though.
considered as
surgery.
☆─────────────────────────────────────☆
Aplusplus (Hakuna Matata) 于 (Wed Apr 13 20:22:42 2011, 美东) 提到:
Just see it.
This reminds me a similar case: A very slim girl about 12 yrs old has
pseudotumor.
☆─────────────────────────────────────☆
Aplusplus (Hakuna Matata) 于 (Wed Apr 13 20:51:25 2011, 美东) 提到:
This case is tough. 20 WB for the first. (from BZ)
A 35 yrs old WF, was sent to ER by her husband in AM since he could not wake
her up.
The patient was entirely normal before last dinner. They planned to have
BBQ, but after she drank a can of beer and 3 cans of soda, she felt sick. So
she went to bed directly.
At ER, she was stuporous, a drug screen was neg. CT of brain, EKG, CXR were
all normal. LP is neg. UA(-), Blood culture was sent. All lab was normal
except Glucose was 60. So she was given D5 1/2NS and tranferred to ICU.
When she arrived ICU, she was comatose and subsequetly she was intubated.
PMH. Healthy except Gastric bypass surgery 6 month ago.
Social: No Cig/ETOH abuse, no drug abuse.
FMH: CAD, HTN.
ROS: N/A
PE: BP: 90/60, P: 45-55, Temp: 93.5 F
Gen: Comatose, on vent.
HEENT: Normal.
Neck: supple.
Chest/CVS: Normal except bradycardia.
Abdomen: Normal.
GU: Normal
Skin: No bruises/ Rash.
Neuro: PERRL. (+)Doll's eye.
(+)pain withdrawal bilaterally. Low muscle tone. No jerking.
(-)Babinski.
Workup in ER please refer to history.
A treatment rapidly reversed her sysptoms.
What is your assessment and plan?

☆─────────────────────────────────────☆
yuzaiusa (Expecto Patronum!) 于 (Wed Apr 13 22:05:54 2011, 美东) 提到:
alcohol intoxication?
treated with HD?
☆─────────────────────────────────────☆
Aplusplus (Hakuna Matata) 于 (Wed Apr 13 22:17:04 2011, 美东) 提到:
No, please read the info carefully:
1. She is not alcoholic.
2. Drug screen is negative.
☆─────────────────────────────────────☆
Lexian (蒙古大夫) 于 (Wed Apr 13 22:20:16 2011, 美东) 提到:
Dumping?
☆─────────────────────────────────────☆
daisyy (Daisy) 于 (Wed Apr 13 22:21:50 2011, 美东) 提到:
except Glucose was 60.
Healthy except Gastric bypass surgery 6 month ago.
恩?这病人在减肥?
Chest/CVS: Normal except bradycardia.
难道是beta blocker过量?
Glucagon
☆─────────────────────────────────────☆
snowfox01 (雪山飞狐) 于 (Wed Apr 13 22:27:49 2011, 美东) 提到:
Let me guess,
Dr. Lexian(前辈) almost get the reason. patient who had gastric bypass surgery
will get dumping symptoms, that is why she went to bed within 30~40 min
after take the alcohol. Also after bypass surgery, patient will absorb
alcohol quickly and excrete slowly.
☆─────────────────────────────────────☆
Aplusplus (Hakuna Matata) 于 (Wed Apr 13 22:35:07 2011, 美东) 提到:
You caught a clue.
☆─────────────────────────────────────☆
Aplusplus (Hakuna Matata) 于 (Wed Apr 13 22:36:03 2011, 美东) 提到:
Glucose 60 is borderline.
She is not on any drugs.
☆─────────────────────────────────────☆
snowfox01 (雪山飞狐) 于 (Wed Apr 13 22:37:26 2011, 美东) 提到:
I will go with Yuzaiusa, My guess is: alcohol intoxication and hypoglycemic
shock.
Don't know how to treat the patient.
☆─────────────────────────────────────☆
Aplusplus (Hakuna Matata) 于 (Wed Apr 13 22:39:49 2011, 美东) 提到:
Why hypothermia?
Why Bradycardia?
☆─────────────────────────────────────☆
Aplusplus (Hakuna Matata) 于 (Wed Apr 13 22:41:25 2011, 美东) 提到:
The fact was that she got worse after she received D5 1/2NS.
And I repeat: Drug screen is neg for ETOH. So no intoxication.
hypoglycemic
☆─────────────────────────────────────☆
yuzaiusa (Expecto Patronum!) 于 (Wed Apr 13 22:43:21 2011, 美东) 提到:
What was her ABG?
★ Sent from iPhone App: iReader Mitbbs 6.88 - iPad Lite
☆─────────────────────────────────────☆
Aplusplus (Hakuna Matata) 于 (Wed Apr 13 22:49:56 2011, 美东) 提到:
Normal.
☆─────────────────────────────────────☆
snowfox01 (雪山飞狐) 于 (Wed Apr 13 23:00:01 2011, 美东) 提到:
It must have something to do with the bypass surgery. I know post bypass
surgery can cause B12 deficiency and iron deficiency, not sure about B1
deficiency. May be caused by B1 deficiency and D5 can worsen the symptoms.
☆─────────────────────────────────────☆
daisyy (Daisy) 于 (Wed Apr 13 23:05:03 2011, 美东) 提到:
恩。这个beta blocker虽然使心率减慢,但是降低insulin的释放,是会使血糖升高的
,所以也不合
理。
☆─────────────────────────────────────☆
daisyy (Daisy) 于 (Wed Apr 13 23:09:46 2011, 美东) 提到:
dumping syndrome 糖类进入肠道过快,引起insulin释放过多。 补糖水的话insulin就
继续增
高?
octreotide 可以降低insulin的释放?
☆─────────────────────────────────────☆
Aplusplus (Hakuna Matata) 于 (Wed Apr 13 23:10:38 2011, 美东) 提到:
Why hypothermia?You never think about sepsis?
☆─────────────────────────────────────☆
Lexian (蒙古大夫) 于 (Wed Apr 13 23:12:54 2011, 美东) 提到:
快抢碘盐。
☆─────────────────────────────────────☆
Aplusplus (Hakuna Matata) 于 (Wed Apr 13 23:14:12 2011, 美东) 提到:
Her Glucose had been normal since D5 was given.
☆─────────────────────────────────────☆
Aplusplus (Hakuna Matata) 于 (Wed Apr 13 23:14:52 2011, 美东) 提到:
LOL!
☆─────────────────────────────────────☆
snowfox01 (雪山飞狐) 于 (Wed Apr 13 23:15:08 2011, 美东) 提到:
Blind loop syndrome?
☆─────────────────────────────────────☆
Aplusplus (Hakuna Matata) 于 (Wed Apr 13 23:16:13 2011, 美东) 提到:
Good thought, but why?
Can you explain why bradycardia?
☆─────────────────────────────────────☆
snowfox01 (雪山飞狐) 于 (Wed Apr 13 23:26:26 2011, 美东) 提到:
Since you mentioned sepsis, blind loop syndrome can cause bacteria over grow.
But I still vote for my B1 deficiency caused by post bypass and alcohol and
D5 consumption.
☆─────────────────────────────────────☆
Aplusplus (Hakuna Matata) 于 (Wed Apr 13 23:31:07 2011, 美东) 提到:
So what do you call this disease? And what do you do?
grow.
and
☆─────────────────────────────────────☆
daisyy (Daisy) 于 (Wed Apr 13 23:32:44 2011, 美东) 提到:
如果是sepsis的话那不是要用antibiotics?
☆─────────────────────────────────────☆
Aplusplus (Hakuna Matata) 于 (Wed Apr 13 23:36:10 2011, 美东) 提到:
You are right. She was treated with antibiotics. Sepsis can present as
hypothermia.
☆─────────────────────────────────────☆
snowfox01 (雪山飞狐) 于 (Wed Apr 13 23:41:02 2011, 美东) 提到:
Thanks for the case.
☆─────────────────────────────────────☆
daisyy (Daisy) 于 (Wed Apr 13 23:48:00 2011, 美东) 提到:
她这是很严重的sepsis了。都快septic shock了吧。 hypotention(borderline
though), hypothermia,
bradycardia, 再不及时可能DIC都出来了。
请教前辈象她这种开始没有发热进展迅速的sepsis多见吗? 有个问题,给糖盐水应该
也没有加重她的
病情,只是病情没有得到控制发展的吧?
☆─────────────────────────────────────☆
daisyy (Daisy) 于 (Wed Apr 13 23:52:39 2011, 美东) 提到:
A very good case. Thanks so much!
☆─────────────────────────────────────☆
Aplusplus (Hakuna Matata) 于 (Wed Apr 13 23:55:25 2011, 美东) 提到:
You give up?
The final diagnosis is : Wernicke's Encephalopathy.
Treatment: Thiamine 100 mg iv stat.
Patient dramatically improved after the injection. Her Vitamin B1 level
came back 1 wk later and proved the diagnosis.
Why bradycardia and hypotension, hypothermia? Hypothalamas involvement. But
you need to memorize mammillary body first.
Risk factors? Gastric bypass, poor nutrition, Beer consumption, and giving
D5 without thiamine!
This is not sepsis. Blood culture was negative.
I think Snowfox01 deserves the prize.
Good night.
☆─────────────────────────────────────☆
skyscorpio (天之蝎子) 于 (Wed Apr 13 23:58:39 2011, 美东) 提到:
谢谢!
我其实一眼就想到了 LOL 包子很快发出
But
giving
☆─────────────────────────────────────☆
daisyy (Daisy) 于 (Thu Apr 14 00:01:50 2011, 美东) 提到:
Haha. This makes sense.
I was fooled by your comments.
But
giving
☆─────────────────────────────────────☆
snowfox01 (雪山飞狐) 于 (Thu Apr 14 00:02:32 2011, 美东) 提到:
谢,前辈。
哎,我总赶不上最后一步。“当机遇像雨点般向我洒来,我都镇定自若的一一闪过”
But
giving
☆─────────────────────────────────────☆
snowfox01 (雪山飞狐) 于 (Thu Apr 14 00:03:49 2011, 美东) 提到:
谢, 包子。
☆─────────────────────────────────────☆
Aplusplus (Hakuna Matata) 于 (Thu Apr 14 00:11:35 2011, 美东) 提到:
我早就看出来了, 知道答案的都潜水!
BZ我发。
☆─────────────────────────────────────☆
Aplusplus (Hakuna Matata) 于 (Thu Apr 14 00:13:50 2011, 美东) 提到:
你跟蝎子一样, 美女半夜敲门,你们在门后加锁。
☆─────────────────────────────────────☆
skyscorpio (天之蝎子) 于 (Thu Apr 14 00:14:09 2011, 美东) 提到:
呵呵,您有时间再出一个吧,下回我跳出来献献丑。
☆─────────────────────────────────────☆
Aplusplus (Hakuna Matata) 于 (Thu Apr 14 00:15:02 2011, 美东) 提到:
跟着感觉走。
☆─────────────────────────────────────☆
daisyy (Daisy) 于 (Thu Apr 14 10:32:14 2011, 美东) 提到:
From Wiki:
Thiamine is released by the action of phosphatase and pyrophosphatase in
the upper small intestine. At low concentrations, the process is carrier
mediated and at higher concentrations, absorption occurs via passive
diffusion. Active transport is greatest in the jejunum and ileum (it is
inhibited by alcohol consumption and by folic deficiency).
symptoms.
☆─────────────────────────────────────☆
susufairy (susufairy) 于 (Thu Apr 14 17:16:19 2011, 美东) 提到:
来说个比较特别的例子,谁都没诊断出来而且病人死了。当时在国内一医院当实习医生
见到的,所以不好用英文,一孕39周产妇入院,入院时已开4指,并伴有宫缩疼(注意
,很可能不一定是宫缩疼,我后面会说),入院后生命体征一切正常除了腹疼,偶伴有
恶心,未吐。所有人都认为是宫缩疼,因为开指非常缓慢,记不清多长时间,最终产妇
于第二天行C-SECTION,手术也还顺利,只是手术过程中发现病人血液呈紫色,腹腔中
有小量血性积液。术后两天病人仍腹痛,且有低热,当时考虑是可能术后感染和术后疼,因为是实习医生,再后来做了什么检查就不记得了,后病人诊断为妊辰合并急性坏死性胰腺炎
,病人于C-SECTION后5天死亡,可怜那只有几天的宝宝就这样失去了妈妈。
☆─────────────────────────────────────☆
Aplusplus (Hakuna Matata) 于 (Thu Apr 14 17:41:06 2011, 美东) 提到:
Sad story.
That's why physician should never be dogmatic. Pelvic pain and abdominal
pain are different but if a physician never had the idea that acute
pancreatitis can be a complication of pregancy, he/she just would be
clueless like the case.
Thanks, susufairy, for your contribution. BZ for you. (contributed by bZ
and Snowfox)
Reasons for Acute Pancreatitis and Pregnancy
Most cases of acute pancreatitis in pregnancy are caused by gallstone
disease. It is thought with the weight and hormonal changes induced by
pregnancy, gallstones are more likely to form and thus travel down the
common bile duct to obstruct the pancreas duct outflow. Another proposed
mechanism for acute pancreatitis in pregnancy is high fat levels in the
blood called triglycerides. Again, the hormonal changes of pregnancy can
predispose certain women to developing this condition. When the
triglyceride levels become too high, oxygen cannot adequately travel to the
pancreas via the bloodstream, and pancreatitis can ensue. Of course, all of
the other reasons for developing acute pancreatitis – alcohol use,
reaction to certain medications, trauma to the pancreatic duct – can also
lead to acute pancreatitis in pregnancy
疼,因为是实习医生,再后来做了什么检查就不记得了,后病人诊断为妊辰合并急性坏
死性胰腺炎
☆─────────────────────────────────────☆
yuzaiusa (Expecto Patronum!) 于 (Thu Apr 14 18:56:47 2011, 美东) 提到:
interesting case. I thought EtOH intoxication can suppress the resp. center
, cause hypoxia, then bradycardia. metabolic acidosis can cause bradycardia
and hypotension too. Never thought about vitamin deficiency. Thanks.
But
giving
☆─────────────────────────────────────☆
Aplusplus (Hakuna Matata) 于 (Thu Apr 14 20:06:43 2011, 美东) 提到:
This tough case was mishandled at multiple steps.
At ER, she was found to be hypoglycemic which is very common to cause MS
changes, so she got the D5 without thiamine which made her worse.
Second, she developped hypothermia which led to a diagnosis of Sepsis.
Bradycardia and hypotension were thought to be septic shock.
Fortunately, I took over the case and made the diagnosis after I had a long
talk with her husband.
She hardly had any residual symptoms at the discharge. But if untreated, she
would end up with Korsakoff's syndrome if she survives.
center
bradycardia
☆─────────────────────────────────────☆
Lexian (蒙古大夫) 于 (Thu Apr 14 21:08:05 2011, 美东) 提到:
A big thank you, A , i think everyone joined this discussion wont forget
Wernicke's encephalopathy from now on.
had u seen one in training before this case? I only read about in textbook,
always thought it's a chronic presentation.
This tough case was mishandled at multiple steps.At ER, she was found to be
hypoglycemic........
★ Sent from iPhone App: iReader Mitbbs 6.88 - iPhone Lite
☆─────────────────────────────────────☆
Aplusplus (Hakuna Matata) 于 (Thu Apr 14 21:18:42 2011, 美东) 提到:
Be honest with you, I did not.
But I know, chances favor prepared mind, lol.
,
be
☆─────────────────────────────────────☆
Lexian (蒙古大夫) 于 (Thu Apr 14 21:25:54 2011, 美东) 提到:
I'm sure your internal reward from this case far outweight your 1500 bill,
lol. Hats off to u.
Be honest with you, I did not. But I know, chances favor prepared mind, lol.
★ Sent from iPhone App: iReader Mitbbs 6.88 - iPhone Lite
☆─────────────────────────────────────☆
Aplusplus (Hakuna Matata) 于 (Thu Apr 14 22:25:24 2011, 美东) 提到:
你来个麻醉case? 讲讲张春桥妹妹被laughing gas谋杀之事?
lol.
☆─────────────────────────────────────☆
Lexian (蒙古大夫) 于 (Thu Apr 14 23:28:14 2011, 美东) 提到:
我做的主要是操作, 诊断大多很简单, 就是要on alert, 出状况时反应要快, 处理
要及时。
几年前在一个inner city的trauma center值班,急诊来了个举枪自杀的小伙子, 据说
为情所困, 大概太激愤了的缘故,开枪时头后仰的厉害。 子弹从下巴射入, 居然从
右眼出来。 大脑没事。 送来时半边脸肿成猪头, 嘴里从咽喉以上一片血糊。 ems插
管插不进去。 我操了根纤维镜顺着有气泡的方向往里送, 居然找到气管里。 管插好
了, 其它很快就稳定下来了。 这大概是internal reward最大的case了。呵呵。
☆─────────────────────────────────────☆
Aplusplus (Hakuna Matata) 于 (Thu Apr 14 23:43:31 2011, 美东) 提到:
我最最佩服的是你们插管和腰穿的技巧。仰视。。。
☆─────────────────────────────────────☆
blulue (小黑) 于 (Fri Apr 15 02:16:32 2011, 美东) 提到:
But
giving
相当好的帖子阿,一直跟着看,学到很多东西!!
看了解释,我有几个没想明白的点,自己搜了搜答案,还是有些地方不太明白,想向
Aplusplus前辈请教:
"Wernicke-Korsakoff syndrome results from severe acute deficiency
superimposed on chronic deficiency."
1,Wernicke的表现?考试时候背的classic triad of oculomotor abnormalities,
ataxia, and confusion?
原来经典三联征齐全的,只有20%的病人.这个病人影响到hypothalamas,低压低温.
还有病人影响到thalamas,progressing hearing loss.
2,Chronic deficiency何来?gastric bypass surgery 6 months ago.
B/C Bypass partial small intestine--defect absorption. In early post op
period,difficult to eat enough for enough VitB1.
Google到两个bypass sugery以后VitB1缺乏致病的,一个术后5个月,一个术后4个月.
3,severe acute deficiency 何来?
是beer consumption吗?她没饮酒史,alcoholism造成chronic deficiency算不上.还是
ER补糖不补B1?感觉更像后者...那beer在这起来神马作用呢?
抱歉写得罗里罗嗦,先谢谢前辈!
☆─────────────────────────────────────☆
Aplusplus (Hakuna Matata) 于 (Fri Apr 15 10:36:03 2011, 美东) 提到:
1,Wernicke的表现?考试时候背的classic triad of oculomotor abnormalities,
ataxia, and confusion?
原来经典三联征齐全的,只有20%的病人.这个病人影响到hypothalamas,低压低温.
还有病人影响到thalamas,progressing hearing loss.
-----------------------------------------
Should be hypothalamus, my bad.
This patient was in coma, so ocular symptoms and gait could not be assessed
accurately.
2,Chronic deficiency何来?gastric bypass surgery 6 months ago.
B/C Bypass partial small intestine--defect absorption. In early post op
period,difficult to eat enough for enough VitB1.
Google到两个bypass sugery以后VitB1缺乏致病的,一个术后5个月,一个术后4个月.
-----------------------------------------
Social history is hard to get and never to be reliable. This patient has
history of bypass, and she does drink Beers.
3,severe acute deficiency 何来?
是beer consumption吗?她没饮酒史,alcoholism造成chronic deficiency算不上.还是
ER补糖不补B1?感觉更像后者...那beer在这起来神马作用呢?
-------------------------
Once again, she drinks beers, that's enough. All drinkers are liars,
believe or not. But I can not say she is alcoholic if she or her husband
denies.
ETOH is a big problem for the society.
Thanks for the interest.
☆─────────────────────────────────────☆
gg2 (Allison) 于 (Fri Apr 15 11:28:25 2011, 美东) 提到:
thanks! learned a lot from these case discussions. please keep it on.
☆─────────────────────────────────────☆
blulue (小黑) 于 (Fri Apr 15 12:35:38 2011, 美东) 提到:
assessed
多谢多谢前辈!!
书本跟临床,结合起来学,收获才更大啊!
☆─────────────────────────────────────☆
REDpersimmon (大尾巴睡猫) 于 (Fri Apr 15 15:52:43 2011, 美东) 提到:
大赞此帖!多谢前辈,您让我头一次感受到原来internal medicine也可以这么有趣!
☆─────────────────────────────────────☆
Aplusplus (Hakuna Matata) 于 (Fri Apr 15 16:17:50 2011, 美东) 提到:
Thanks all above for my internal rewards, LOL.
☆─────────────────────────────────────☆
skyscorpio (天之蝎子) 于 (Fri Apr 15 16:28:53 2011, 美东) 提到:
you are great! much better than Dr. Lexain LOL
☆─────────────────────────────────────☆
Aplusplus (Hakuna Matata) 于 (Fri Apr 15 16:42:21 2011, 美东) 提到:
I feel chills. LOL
☆─────────────────────────────────────☆
Lexian (蒙古大夫) 于 (Fri Apr 15 17:10:35 2011, 美东) 提到:
I got the internal reward of keeping A+ here, which is not less than anything
else.
☆─────────────────────────────────────☆
RBC120 (LTC) 于 (Fri Apr 15 20:46:51 2011, 美东) 提到:
在产前检查门诊实习,第一次用听筒在两个不同部位听到胎心,确诊为双胞胎!孕妇高
兴,我也很有成就感。
☆─────────────────────────────────────☆
VictorG (VictorG) 于 (Sat Apr 16 09:12:39 2011, 美东) 提到:
Good case, progressing so fast.
lymphocytes
no
the
☆─────────────────────────────────────☆
VictorG (VictorG) 于 (Sat Apr 16 09:21:24 2011, 美东) 提到:
Narcotics withdrawal----confusion, interesting.
he
☆─────────────────────────────────────☆
VictorG (VictorG) 于 (Sat Apr 16 09:52:23 2011, 美东) 提到:
鹅买高的。
But
giving
☆─────────────────────────────────────☆
VictorG (VictorG) 于 (Sat Apr 16 10:13:40 2011, 美东) 提到:
看了大家的,收获颇丰,也贡献一个,如果算的话。
我有一个CASE, 需要诊断。
一个朋友,N年前买了炸鸡腿,早餐吃了两口,味有点怪,一点都不香,就不吃了。3个
小时以后恶心,呕吐,很快出现眩晕(VERTIGO)很严重的眩晕,无法站立行走,持续
恶心呕吐眩晕,症状持续6个小时左右缓解。客观原因(just started new job)没去医
院(后来朋友后怕,食物中毒,应该去).
因为最后诊断不明确是何种食物中毒造成这么严重的眩晕,希望大家看看。
☆─────────────────────────────────────☆
Aplusplus (Hakuna Matata) 于 (Sat Apr 16 11:49:21 2011, 美东) 提到:
Wallenburg's syndrome.
Your friend had a TIA or stroke in the brain stem. I guess he is a middle
age or older man and has stroke risk factors. Otherwise, I could be wrong.
大家如果觉得我说得有道理, 就复习一下Wallenburg's syndrome吧。 我在医学院神
经科实习时, 就记得这个case, 其他都忘了, lol。
“一个朋友,N年前买了炸鸡腿,早餐吃了两口,味有点怪,一点都不香,就不吃了”
Taste changes
"3个小时以后恶心,呕吐,很快出现眩晕(VERTIGO)很严重的眩晕,无法站立行走,持续
Symptoms exacerbated and involved more area of medulla.
"症状持续6个小时左右缓解"
Possible TIA only.
☆─────────────────────────────────────☆
VictorG (VictorG) 于 (Sat Apr 16 12:13:47 2011, 美东) 提到:
如果是中老年,有危险因素,我想这个肯定top on differential.
那一次发生时朋友很年轻(<30 years old),之后多年也没有过任何illness, 食物中
毒应该是准确的,只是拿不准是那种中毒。
我查了一下,肉毒毒素(clostridium toxin)有可能,可导致vertigo,好像当时也有
weakness, 忘了告诉大家了。
如果是的话,还是很危险的 nausea, vomitting, vertigo, muscle weakness.
,持续
☆─────────────────────────────────────☆
VictorG (VictorG) 于 (Sat Apr 16 12:23:56 2011, 美东) 提到:
再一次说明病例描述病史很重要,忘了病例的年龄,伴发症状,有无其他危险因素以及
其他情况等等。没有这些,看来呈现的完全是另外一种情况。
这个病例,发病是<30 years old, 平素健康,无任何疾病。之后多年也无任何疾病。
☆─────────────────────────────────────☆
daisyy (Daisy) 于 (Sat Apr 16 18:36:10 2011, 美东) 提到:
With such short onset time (3 hours after the food intake), I think it's
most likely caused by preformed toxins produced by bacteria. The
symptoms were gone after 6 hours also support food poison by preformed
toxins.
The most common one is Staphylococcus aureus. The other one is Bacillus
cereus. BC usually be found in reheated rice, so it's unlikely the
culprit in this case.
S.Aureus can be found in meat, mayonnaise, custard. So I think S. Aureus
preformed toxin may be the most likely one.
Pathogenesis: Enterotoxin acts on receptors in gut that transmit
impulses to medullary centers.
Nausea, vomiting, abdominal pain are common symptoms in S. Aureus food
poison. Although vertigo is not a common one, but I think the overall
profile of the case still fits S.Aureus better.
☆─────────────────────────────────────☆
daisyy (Daisy) 于 (Sat Apr 16 18:47:48 2011, 美东) 提到:
Clostridium Botulinum toxin was found in Canned foods (eg, smoked fish,
mushrooms, vegetables, honey).
Presentation:
Descending weakness and paralysis start 1-4 days after ingestion, followed
by constipation.
Mortality is very high.
http://emedicine.medscape.com/article/175569-clinical#a0218
☆─────────────────────────────────────☆
Aplusplus (Hakuna Matata) 于 (Sat Apr 16 22:31:47 2011, 美东) 提到:
Very well summarized, impressive!
Botulinum toxin causes descending paralysis started with diplopia,
dysartheria or dysphagia and facial paralysis. Vertigo is rare.
I agree with you on bacteria toxin. More common symptoms are N/V ,abdominal
pain and diarrhea. Dizziness is common but not vertigo.
When patient says dizzy, make sure if patient means lightheadedness or
vertigo. Big difference.
☆─────────────────────────────────────☆
daisyy (Daisy) 于 (Sat Apr 16 23:29:30 2011, 美东) 提到:
Totally agree with you!
Dizziness is common but not vertigo.
☆─────────────────────────────────────☆
rainingcats (喵喵) 于 (Sun Apr 17 01:59:23 2011, 美东) 提到:
我有过同样的经历啊,朋友送的一盒蛋糕,吃了一个多小时以后恶心,眩晕,站不住,
眼前一黑差点昏过去,吐了才好。
☆─────────────────────────────────────☆
Aplusplus (Hakuna Matata) 于 (Sun Apr 17 11:16:18 2011, 美东) 提到:
所以说呕吐是人体自我保护机制。
☆─────────────────────────────────────☆
rainingcats (喵喵) 于 (Sun Apr 17 11:22:06 2011, 美东) 提到:
为什么吐了以后立刻就好了呢?如果是因为毒素,那么毒素要进入中枢,才会产生头昏
等症状。而呕吐只是清除胃内容物,会避免更多毒素的影响,但不应该影响已经进入血
液的毒素代谢,为什么吐完1秒钟之内立刻缓解,这病理生理机制是什么?
☆─────────────────────────────────────☆
Aplusplus (Hakuna Matata) 于 (Sun Apr 17 11:46:40 2011, 美东) 提到:
我来试试回答你的问题:
1. 清除胃内容物是一部分, 减缓诱因。
2. 更重要的是:呕吐是vagal nerve response。When parasympathetic nerve
overactivated, sympathetic response will kick in. Your body will be under
adrenaline influence. 你的身体处于应激状态, 所以症状会得到暂时的缓解。
当然应激反应很复杂, 就不多说了。
☆─────────────────────────────────────☆
rainingcats (喵喵) 于 (Sun Apr 17 11:59:16 2011, 美东) 提到:
可能是这个原因吧
under
☆─────────────────────────────────────☆
Aplusplus (Hakuna Matata) 于 (Sun Apr 17 22:28:29 2011, 美东) 提到:
40 yrs old WF, presented with bilateral photophobia. She had this type
problems for the past several years and now her symptoms were getting
progressively worse. Her eye exams were reported normal. She had to wear
sunglasses all the time.
She told me at clinic that she had to go early since her daughter has an
endoscopy scheduled for mysterious abdominal pain.
PMH: Fibromyalgia, bipolar disorder, IBS. She also has chronic abdominal
pain and been scoped multiple times.
Social: Smokes 1 ppd; Social drink. No drug abuse.
FMH: Fibromyalgia, IBS.
ROS; Not remarkable except mentioned above.
PE: Vital: Normal.
All other PEs are not remarkable except severe photophobia, overactive BS
and mild diffuse abdominal tenderness.
Dx: Photophobia, unknown etiolgy
Your next step?
☆─────────────────────────────────────☆
skyscorpio (天之蝎子) 于 (Sun Apr 17 23:46:49 2011, 美东) 提到:
urine analysis, looking for something specific
☆─────────────────────────────────────☆
Aplusplus (Hakuna Matata) 于 (Mon Apr 18 08:38:38 2011, 美东) 提到:
Good for you!
You finally knocked the door. lol.
Would you share your google results with us?:}
☆─────────────────────────────────────☆
skyscorpio (天之蝎子) 于 (Mon Apr 18 21:06:05 2011, 美东) 提到:
porphyria caused by uroporphyrinogen decarboxylase deficiency, urine test to
look at uroporphyrinogen level.
☆─────────────────────────────────────☆
skyscorpio (天之蝎子) 于 (Mon Apr 18 21:18:54 2011, 美东) 提到:
包子包子!
LOL
to
☆─────────────────────────────────────☆
Aplusplus (Hakuna Matata) 于 (Mon Apr 18 21:34:40 2011, 美东) 提到:
The history is important. With a family history of IBS, chronic pain, mood
disorder and photosensitivity, please consider porphyria as a DDx.
There are different types of porphyria, and a blood test sometimes gives you
more info. I remember I ordered a bllood test then for the diagnosis.
Porphyria is very under-diagnosed as it can imitate a lot of other
complaints.
Typically, patients have been put through many expensive tests or procedures
before it is thought to check for porphyria. The cost of testing for
porphyria is not high compared to many other tests that are routinely done.
Controversially, the genetic predisposition for porphyria could be as high
as 1 in 500 in the population although active porphyria may be as low as 1
in 10,000.
Porphyria is incurable as yet. There is a high emphasis on prevention
strategies to keep it latent or symptom-free for a long life. But some
porphyrias can become serious progressive degenerative disorders if repeated
attacks are not stopped. Some treatments are available. like beta blockers.
Porphyria has been mistaken for: Guillain Barre, growing pains, Lupus,
eczema, somatisation disorders, epilepsy, MS, dermatology, depression,
appendicitis,Parkinson's and many, many more.
Suspicion is higher if symptoms come and go or appear after medication or
anaesthetic
to
☆─────────────────────────────────────☆
Aplusplus (Hakuna Matata) 于 (Mon Apr 18 21:39:16 2011, 美东) 提到:
The BZs are sponsored by skyscorpio and snowfox, many thanks to them.
☆─────────────────────────────────────☆
Aplusplus (Hakuna Matata) 于 (Sat Apr 23 00:25:49 2011, 美东) 提到:
Just read the case on the web, interesting.
"A woman came to the ER with severe abd pain, had dozens of visits through
the years, multiple specialists and told she had irritable bowel,
colonoscopies and CT in past c/w bowel edema, non-specific at times.
Laparoscopies also negative. Told she was a seeker on many occasions. I saw
her writhing in pain, typical narcs, etc. Repeat CT in ER with mod bowel
edema, also noticed she had a rash looked like hives. I asked, "do you break
out like this all the time these episodes happen."
"I think so." she said. "But they said it was always from the dilaudid or
morphine. and give me benadryl"
I gave her IV FFP in the ER and she completely resolved her rash and pain,
and she said..."OH my GOD, that stuff you gave me worked."
Labs showed severe C1 esterase deficiency. "
Have a nice weekend.
☆─────────────────────────────────────☆
snowfox01 (雪山飞狐) 于 (Sun Apr 24 12:52:46 2011, 美东) 提到:
Thanks for another excellent case.
saw
break
☆─────────────────────────────────────☆
Lexian (蒙古大夫) 于 (Sun Apr 24 20:02:30 2011, 美东) 提到:
This was in my mind yesterday, would actually be a good ethnic question.
OP: http://www.mitbbs.com/article_t0/MedicalCareer/31417961.html
What would you do if the OB doc hasn't made it here? baby's umbilical cord
came out from the birth canal. fetal heart rate shows decel. OB was called
but hasn't got here. You are the only doc available but it's not your
specialty. Do you cut or not?
☆─────────────────────────────────────☆
sunshadow (kageko) 于 (Sun Apr 24 20:22:13 2011, 美东) 提到:
呵呵,我压力大的时候,工作要紧的时候无论时间多久都不能吃东西。每吃必吐。
那种时候只喝水或者果汁。
☆─────────────────────────────────────☆
Aplusplus (Hakuna Matata) 于 (Sun Apr 24 21:34:41 2011, 美东) 提到:
Do not do it.
You could be a hero but you could face big troubles as well.
1. You might have a consent signed by you for anesthesia but not delievering
related procedures.
2. You are not licensed to deliever babys.
It really depended on the outcome. Even you did right thing but bad luck,
you would get into troubles.
Any OB doc here?
☆─────────────────────────────────────☆
Aplusplus (Hakuna Matata) 于 (Sun Apr 24 21:45:52 2011, 美东) 提到:
Your case is different. You are sick due to autonomous dysfunction (high
parasympathetic activity). This is consequence of stress which is adrenaline
related.
So in your case, the compensation for stress gives you the trouble.
What you need to do is:
1. Reduce the stress, of course, easy to say, difficult to do. I believe better sleep is a good way to relieve stress. Take something like melatonin if you do have insomnia.
2. Antiemetics should work. I like promethazine 25 mg po tid prn. and you can enjoy your meal under the stress.
☆─────────────────────────────────────☆
sunshadow (kageko) 于 (Sun Apr 24 21:55:50 2011, 美东) 提到:
啊,居然得到回复了。
那我多说两句,关于吐这个问题,我貌似是易吐体质(绝对不是减肥闹的),这几年才
开始有,压力大的时候吃了会吐,有时候来月经也会痛到吐,情绪激动也有两回,酒吐
那更是常事,汗。
为这个事情被周围的八卦男女怀疑有喜无数次。jiong。
adrenaline
can enjoy your meal under the stress.
☆─────────────────────────────────────☆
snowfox01 (雪山飞狐) 于 (Sun Apr 24 22:07:07 2011, 美东) 提到:
Thanks for the explanation. It was very educational.
delievering related procedures.
you would get into troubles.
☆─────────────────────────────────────☆
Aplusplus (Hakuna Matata) 于 (Sun Apr 24 22:43:50 2011, 美东) 提到:
Your case reminds me the sentance:
No good deed goes unpunished.
Do you know what it means? LOL!
This is the case of tonight, anyone jumps in? BZ available.
☆─────────────────────────────────────☆
Lexian (蒙古大夫) 于 (Sun Apr 24 22:51:14 2011, 美东) 提到:
I'd like to hear what these exam takers say...
☆─────────────────────────────────────☆
Aplusplus (Hakuna Matata) 于 (Sun Apr 24 22:52:15 2011, 美东) 提到:
C'mom, anybody can try except DrNewbie.
☆─────────────────────────────────────☆
Aplusplus (Hakuna Matata) 于 (Sun Apr 24 23:11:27 2011, 美东) 提到:
国内称植物神经忞乱, 不过最好让GI先看一下。
☆─────────────────────────────────────☆
Aplusplus (Hakuna Matata) 于 (Sun Apr 24 23:16:57 2011, 美东) 提到:
OK, I am not going to wait.
Dr.Lexian想做好事, 但很可能因此被sue。 这就叫好心不得好报, 反而被punished。
此所谓 No good deed goes unpunished。
☆─────────────────────────────────────☆
Lexian (蒙古大夫) 于 (Sun Apr 24 23:22:46 2011, 美东) 提到:
punished。
no, i'm afraid "afraid of getting sued" is not the right answer. I'd asked
the hospital risk management attorney about similar situation before. I'll
tell you what lawyer said.
☆─────────────────────────────────────☆
skyscorpio (天之蝎子) 于 (Sun Apr 24 23:29:01 2011, 美东) 提到:
因为不是您的病人,您可以选择做,也可以选择不做。如果您有自信做当然做了。而且,只要是在一个医生应该做的范围内,您的行为不用承担法律责任。
包子拿来!:)
☆─────────────────────────────────────☆
Lexian (蒙古大夫) 于 (Sun Apr 24 23:40:24 2011, 美东) 提到:
承担法律责任。
wrong answer.
☆─────────────────────────────────────☆
skyscorpio (天之蝎子) 于 (Sun Apr 24 23:42:17 2011, 美东) 提到:
因为不是您的病人,您可以选择做,也可以选择不做。如果您做了,而且,只要是在一
个医生应该做的范围内,您的行为不用承担法律责任。
包子拿来!:)
☆─────────────────────────────────────☆
Lexian (蒙古大夫) 于 (Sun Apr 24 23:51:14 2011, 美东) 提到:
do it, or don't, only one choice. and the reason why.
your answer is not correct, my dear BZ.
☆─────────────────────────────────────☆
Aplusplus (Hakuna Matata) 于 (Sun Apr 24 23:51:36 2011, 美东) 提到:
In case of emergency, you may opt to do whatever is needed to save the
infant. The question is how you justify such a procedure. If she is
unconscious, you are ok. But if she is conscious, oral consent is needed.
asked
☆─────────────────────────────────────☆
Lexian (蒙古大夫) 于 (Sun Apr 24 23:54:39 2011, 美东) 提到:
it's extreme emergency. she is crying, and begging "help me".
☆─────────────────────────────────────☆
skyscorpio (天之蝎子) 于 (Sun Apr 24 23:57:40 2011, 美东) 提到:
您老这个问题绝对超出我们的考试范围了,我拒绝羞愧!
☆─────────────────────────────────────☆
skyscorpio (天之蝎子) 于 (Mon Apr 25 00:07:31 2011, 美东) 提到:
不过我的行为学确实比较差,呼唤今年申请的真正的大牛出来
☆─────────────────────────────────────☆
Aplusplus (Hakuna Matata) 于 (Mon Apr 25 00:12:33 2011, 美东) 提到:
In case of emergency, you do not need consent, have nurse as the witness.
But unless you are comfortable about the diagnosis and the procedures(I
think you may have some trainings on OB), I would not recommend you to do it.
This is not CPR everyone has the training.
☆─────────────────────────────────────☆
Lexian (蒙古大夫) 于 (Mon Apr 25 00:13:11 2011, 美东) 提到:
well, i know one resident was FIRED over this issue. it is hard to
understand so don't feel ashamed.
shouldn't do it. because you are putting the mother's life in danger. No
matter how confident you are, you are not trained in C/S, which may be a
simple procedure in a pair of trained hand. It carries considerable amount
of risk, even life threatening for a untrained person. Should never overlook
mother's life for the baby's.
One of my junior resident was fired because of this. When he was on call for
OB, an emergency happened, OB doc rushed patient to operating room, demand
him (a 2nd yr resident) to put the patient to sleep so a C/S can be done. He
called attending but attending hasn't got here. He went ahead put the
patient to sleep, and the baby was delivered fine. But the attending refused
to sign the chart. This is the lesson we all learned - putting a pregnant
women to sleep carries enormous risk. You can't put baby's interest ahead of
the mother's.
☆─────────────────────────────────────☆
Aplusplus (Hakuna Matata) 于 (Mon Apr 25 00:17:40 2011, 美东) 提到:
That is what i have said. lol.
overlook
for
demand
He
☆─────────────────────────────────────☆
Lexian (蒙古大夫) 于 (Mon Apr 25 00:21:03 2011, 美东) 提到:
yes, but the lawyers can make it sound totally different, right?
☆─────────────────────────────────────☆
Aplusplus (Hakuna Matata) 于 (Mon Apr 25 10:26:41 2011, 美东) 提到:
Mecical practice has been dictated by these morons, sadly.
☆─────────────────────────────────────☆
REDpersimmon (大尾巴睡猫) 于 (Mon Apr 25 10:56:53 2011, 美东) 提到:
the 2nd year resident was in such bad luck, although he might have saved the
baby's life.
This lesson tells us--risk a baby's life and wait for attendings.
i feel the law needs to be changed--what if the 2nd year anethes. resident
was well trained on this and felt confident to do it without risking the
mother's life? Isn't that the patient's autonomy rules everything if the
patient is competent and conscious? What if the mother is willing to risk
her own life in order to save the baby?
☆─────────────────────────────────────☆
Lexian (蒙古大夫) 于 (Mon Apr 25 11:13:43 2011, 美东) 提到:
the
a good question. That would require an advance medical directive legally
constructed ahead of time.
as to the resident, it's not you feel you are confident or not, a resident
has only limited scope to practice medicine under the attending's medical
direction. They are not at the position to judge someone (including
themselves) qualify or not.
☆─────────────────────────────────────☆
snowfox01 (雪山飞狐) 于 (Mon Apr 25 11:28:40 2011, 美东) 提到:
"One of my junior resident was fired because of this. When he was on call
for OB, an emergency happened, OB doc rushed patient to operating room,
demand him (a 2nd yr resident) to put the patient to sleep so a C/S can be
done. He called attending but attending hasn't got here."
I have a question.
what should the poor resident do? there was a emergency and the surgeon
already gave him the order. Can he just tell the OB that I need to wait for
my Attending? and if something happen during the short waiting period, who
will be blamed ?
☆─────────────────────────────────────☆
REDpersimmon (大尾巴睡猫) 于 (Mon Apr 25 11:50:26 2011, 美东) 提到:
Thanks for answering my questions. So let's clarify that---All attendings,
of OB/GYN or Anesthesiology or any specialty--do not order anything out of
scope to us poor residents---as we are not physicians, we are physicians in
training! I'll never try to help anyone in public in those situations during
my residency since we're excluded from good samaritan law as we are not
Physicians! Is that right?
☆─────────────────────────────────────☆
REDpersimmon (大尾巴睡猫) 于 (Mon Apr 25 11:53:54 2011, 美东) 提到:
I have the same question. The resident seemed to be dead anyway.
It's not fair to him especially when the outcome is good. No one is sued,
why punish the resident for such a dilemma? Just warning is enough.
If the baby was dead by the time the attending showed up, I'm sure the
resident was the one to blame, at least by the OB
for
☆─────────────────────────────────────☆
Lexian (蒙古大夫) 于 (Mon Apr 25 14:14:10 2011, 美东) 提到:
He should try to get the approval from the attending (over the phone) first.
If the attending tell him to wait, he should wait. If the attending tell
him to go ahead, then go ahead. The attending is ultimately responsible for
the consequences. If you proceed without prior approval, and then you will
be liable.
for
☆─────────────────────────────────────☆
sunshadow (kageko) 于 (Mon Apr 25 17:01:13 2011, 美东) 提到:
But can Good Samaritan Law be applied here?
first.
for
☆─────────────────────────────────────☆
sunshadow (kageko) 于 (Mon Apr 25 17:04:40 2011, 美东) 提到:
好。谢谢。
☆─────────────────────────────────────☆
snowfox01 (雪山飞狐) 于 (Mon Apr 25 17:05:55 2011, 美东) 提到:
多谢,前辈的教育帖。
first.
for
☆─────────────────────────────────────☆
Lexian (蒙古大夫) 于 (Mon Apr 25 19:24:30 2011, 美东) 提到:
Good Samaritan Law doesn't apply here, you should not endanger the mother
while trying to help the baby, that is the different.
☆─────────────────────────────────────☆
yuzaiusa (Expecto Patronum!) 于 (Tue Apr 26 21:49:59 2011, 美东) 提到:
Good case. Thank u. BTW, are u a neurologist?
40 yrs old WF, presented with bilateral photophobia.
★ Sent from iPhone App: iReader Mitbbs 6.88 - iPad Lite
☆─────────────────────────────────────☆
daisyy (Daisy) 于 (Tue Apr 26 22:06:19 2011, 美东) 提到:
The resident get paid for the activities he performed in the hospital,
Good Samaritan Law can't be applied.
☆─────────────────────────────────────☆
yuzaiusa (Expecto Patronum!) 于 (Tue Apr 26 22:07:04 2011, 美东) 提到:
Here is something I always tell my interns: as a resident, we are
practicing under attending's license, maybe you don't agree with him/her,
you can discuss with them, but you should follow their decisions.
Thanks for answering my questions. So let's clarify that---All attendings,
of OB/GYN or ........
★ Sent from iPhone App: iReader Mitbbs 6.88 - iPad Lite
☆─────────────────────────────────────☆
sunshadow (kageko) 于 (Wed Apr 27 01:11:56 2011, 美东) 提到:
I wasnt talking about that intern. I was talking about Dr. Lexian's
imaginary case.
OB didn't show up, you were there for anesthesia. Pregger asked for your
help.
Can u do it w/o getting punished? Can GSL be applied here?
I guess I should ask my JD friends.
☆─────────────────────────────────────☆
REDpersimmon (大尾巴睡猫) 于 (Wed Apr 27 07:11:44 2011, 美东) 提到:
I did not mean to apply good samaritan law in this particular situation. I
meant in public, as a resident, we should not offer help because we are non-
physicians, and the law only protects real physicians.
☆─────────────────────────────────────☆
REDpersimmon (大尾巴睡猫) 于 (Wed Apr 27 07:13:31 2011, 美东) 提到:
This makes sense. Thanks!
We have a lot to learn about when to 请示。
first.
for
☆─────────────────────────────────────☆
Aplusplus (Hakuna Matata) 于 (Wed Apr 27 21:46:07 2011, 美东) 提到:
Today I got a phone call from a collegue. His previous nurse who is 54
years old female, presented with right foot pain for 4 days. The pain was
so severe that she could not walk. He did a MRI of L-spine and vascular u/s
which were all normal. ESR is 30. He wanted me take a 2nd look if he missed
something.
PMH. Lung Ca for 2 yrs. DM. HTN. Depression.
SH/FH/ROS: not remarkable.
What are you going to do at Exam if you suspect something?
☆─────────────────────────────────────☆
daisyy (Daisy) 于 (Wed Apr 27 22:08:53 2011, 美东) 提到:
In the hospital setting, I think it's hard to apply Good Samaritan Law
because lots of qualified professionals are around.
Try to ask other qualified doctors to help.
☆─────────────────────────────────────☆
sunshadow (kageko) 于 (Wed Apr 27 22:37:31 2011, 美东) 提到:
It makes sense, but still...
Curious to see the real cases.
Will get back if I find some n share w u
☆─────────────────────────────────────☆
daisyy (Daisy) 于 (Wed Apr 27 23:10:14 2011, 美东) 提到:
That will be very interesting.
☆─────────────────────────────────────☆
Aplusplus (Hakuna Matata) 于 (Wed Apr 27 23:47:21 2011, 美东) 提到:
Real cases?
For visitors, send to ER, maybe CPR if pulseless.
For patients, call code。
For Dr.lexien's case, do not do it. Call for help.
☆─────────────────────────────────────☆
dojo (麦地里的豆角-MS0) 于 (Thu Apr 28 00:19:18 2011, 美东) 提到:
Surprised no one gave a try yet.
DM -> peri neuropathy -> neuro exam of R foot, e.g. two-point discrim.
/s
missed
☆─────────────────────────────────────☆
Aplusplus (Hakuna Matata) 于 (Thu Apr 28 08:41:35 2011, 美东) 提到:
Good try.
Her right foot was so touch sensitive that a light touch triggered severe
pain. (allodynia) But her left foot was entirely normal.
The apperance of the foot looked same as the other foot.
☆─────────────────────────────────────☆
Aplusplus (Hakuna Matata) 于 (Thu Apr 28 17:18:15 2011, 美东) 提到:
Still no more people to try?
How about this picture?
/s
missed
☆─────────────────────────────────────☆
snowfox01 (雪山飞狐) 于 (Thu Apr 28 18:35:18 2011, 美东) 提到:
1.我的第一印象是:肺腺癌导致高钙血症---骨折(2年正好是该出现骨破坏).
2.第二印象是:小细胞癌化疗导致的副作用---神经痛。(小细胞癌活到2年?不容易)
3.看了你的图片----猜是癌症导致的免疫低下,引起的带状疱疹感染。
☆─────────────────────────────────────☆
Aplusplus (Hakuna Matata) 于 (Thu Apr 28 19:21:34 2011, 美东) 提到:
The first thing I did was to do a thorough skin exam. I found the rash on
the right buttock just like the pic. So the diagnosis was herpes zoster
radiculopathy.
Anyone got the shinges before? I got once at age 22, it was so painful as
hell!
Her oncologist saw her on same day and called me: " I have been seeing this
all the time".
☆─────────────────────────────────────☆
Aplusplus (Hakuna Matata) 于 (Mon May 2 23:18:22 2011, 美东) 提到:
I did one month OB prior to internal medicine. I remembered there was a
patient with FOU. After the rounds, the attending brought me back to the
patient's room. He said "I'm just afraid I missed something." Then he put
on the gloves and did a thorough skin exam including all the skin folds and
grooves. You know what? He did find a small decubitus ulcer at the bottom
of right buttock. I was not following that case but I really admired his
attitude.
I wish I could find the problem by myself. Since then, I have been so
careful on PEs and one time an ICU attending commented on my PE as
meticulous. I was told by my chairman afterwards.
☆─────────────────────────────────────☆
aoisora (简单就好) 于 (Tue May 3 18:45:17 2011, 美东) 提到:
what is fuo?
☆─────────────────────────────────────☆
aoisora (简单就好) 于 (Tue May 3 18:46:43 2011, 美东) 提到:
sorry about the typo, I meant fou in your posting. thanks.
☆─────────────────────────────────────☆
Aplusplus (Hakuna Matata) 于 (Tue May 3 19:51:52 2011, 美东) 提到:
Fever of Unknown Origin
s********o
发帖数: 3319
2
这是A++前辈在麦地最有影响的一个帖子,可以说是他在买卖提挖到的第一桶金。现在转给新版, 做个永久纪念。版二可以考虑收入精华区.
再次向A++前辈致谢!
A*******s
发帖数: 9638
3
你喜欢吃蝎子的那张照片吗?
谢谢蝎子版主。

【在 s********o 的大作中提到】
: 这是A++前辈在麦地最有影响的一个帖子,可以说是他在买卖提挖到的第一桶金。现在转给新版, 做个永久纪念。版二可以考虑收入精华区.
: 再次向A++前辈致谢!

s********o
发帖数: 3319
4
呵呵,我还记得我刚上台时您开口就叫我“毒蝎”。咱们都应该庆幸,我当时忍住了没有骂还,否则江湖中就会少了一个门派,多了一段恩怨和许多冤魂,LOL
我要淡出江湖一阵子了。您老好好干。如果觉得麦地版有什么帖子你们比较感兴趣可以自己转过来讨论,像很多求医问药的帖子等等。

【在 A*******s 的大作中提到】
: 你喜欢吃蝎子的那张照片吗?
: 谢谢蝎子版主。

A*******s
发帖数: 9638
5
No sweat, LOL!

没有骂还,否则江湖中就会少了一个门派,多了一段恩怨和许多冤魂,LOL
以自己转过来讨论,像很多求医问药的帖子等等。

【在 s********o 的大作中提到】
: 呵呵,我还记得我刚上台时您开口就叫我“毒蝎”。咱们都应该庆幸,我当时忍住了没有骂还,否则江湖中就会少了一个门派,多了一段恩怨和许多冤魂,LOL
: 我要淡出江湖一阵子了。您老好好干。如果觉得麦地版有什么帖子你们比较感兴趣可以自己转过来讨论,像很多求医问药的帖子等等。

A*******s
发帖数: 9638
6
今天下午看到一个case, 与大家分享:
67 yrs old WF with a history of A-fib, presented to ER with diaphoresis and
fatigue for 2 days. She has been on Pradaxa for a-fib. Cardiology suspected CHF exacerbation.
PMH: CAD, CHF and a-fib. No DM.
Social/FMH/ROS: Not remarkable.
On exam, Vital was stable. HR was only 76.
She was noticed to have an anisocoria. She blamed her cataract surgery 10 years ago for the cause of unequal sized pupils. But her family believed it was new onet.
Pupil OS 7 mm, Sluggish to light. OD 3 mm, reactive. ocular movement was
normal. Had wet skins and no reflexes.
All other PEs were not remarkable. Lung was clear. No peripheral edema.
Brain MRI was normal.
What did she have?
See the attached photo below.



在转给新版, 做个永久纪念。版二可以考虑收入精华区.

【在 s********o 的大作中提到】
: 这是A++前辈在麦地最有影响的一个帖子,可以说是他在买卖提挖到的第一桶金。现在转给新版, 做个永久纪念。版二可以考虑收入精华区.
: 再次向A++前辈致谢!

V*****G
发帖数: 337
7
Adie Pupil? Or medication effect?

and
suspected CHF exacerbation.
years ago for the cause of unequal sized pupils. But her family believed it
was new onet.

【在 A*******s 的大作中提到】
: 今天下午看到一个case, 与大家分享:
: 67 yrs old WF with a history of A-fib, presented to ER with diaphoresis and
: fatigue for 2 days. She has been on Pradaxa for a-fib. Cardiology suspected CHF exacerbation.
: PMH: CAD, CHF and a-fib. No DM.
: Social/FMH/ROS: Not remarkable.
: On exam, Vital was stable. HR was only 76.
: She was noticed to have an anisocoria. She blamed her cataract surgery 10 years ago for the cause of unequal sized pupils. But her family believed it was new onet.
: Pupil OS 7 mm, Sluggish to light. OD 3 mm, reactive. ocular movement was
: normal. Had wet skins and no reflexes.
: All other PEs were not remarkable. Lung was clear. No peripheral edema.

A*******s
发帖数: 9638
8
Congratulations!
Adie's syndrome.

it

【在 V*****G 的大作中提到】
: Adie Pupil? Or medication effect?
:
: and
: suspected CHF exacerbation.
: years ago for the cause of unequal sized pupils. But her family believed it
: was new onet.

V*****G
发帖数: 337
9
Thanks for your cases, always can learn more!!!
For that case of food poisoning, I doubt it is staphy aureus, because the
main part of presentation is vertigo. The only thing I find close to that
presentation is Botulism. (I didn't reply at that time is because I cannot
surf often), I really think answer should be Clostridium Botulinum
http://www.livestrong.com/article/111800-botulism-poisoning-sym
Vision Changes
Patients who develop botulism poisoning typically experience vision changes
within 12 to 72 hours after exposure to the contaminated food, explain Food
Safety, a website managed by the U.S. Department of Health and Human
Services. Initially, botulism patients can develop vertigo, which causes
them to feel as though their surroundings are spinning uncontrollably.
Blurred, double or cloudy vision can also arise, making it difficult for
infected patients to see normally. These vision symptoms can contribute to
additional side effects, including dizziness and headache.
Read more: http://www.livestrong.com/article/111800-botulism-poisoning-symptoms/#ixzz1OGlQ71bZ

【在 A*******s 的大作中提到】
: Congratulations!
: Adie's syndrome.
:
: it

V*****G
发帖数: 337
10
I have a real case, just roughly know the history, please bear with me.
A 62 yo w m c/o left shoulder pain for several days
Pain is dull, 6/10,
Caught a cold several days before onset of shoulder pain
With fatigue, slight short of breath
Non smoker
Past history is HTN x 15 years, on diuretics
No travel history
No trauma/injury history
Occupation: office based job
PE: normal
What is the next step?
相关主题
肥胖是一种疾病大家说说, 以你自己的理解, 医学是什么东东?
【病例讨论】AnisocoriaHeparin drip
【有奖征贴】说一说你亲眼见过的最疑难病例新医生上路最常犯的9个错误
进入Medicalpractice版参与讨论
A*******s
发帖数: 9638
11
X-rays of the chest and shoulder.
Dissection?

【在 V*****G 的大作中提到】
: I have a real case, just roughly know the history, please bear with me.
: A 62 yo w m c/o left shoulder pain for several days
: Pain is dull, 6/10,
: Caught a cold several days before onset of shoulder pain
: With fatigue, slight short of breath
: Non smoker
: Past history is HTN x 15 years, on diuretics
: No travel history
: No trauma/injury history
: Occupation: office based job

d****y
发帖数: 2180
12
The pain should be sharp in dissection.

【在 A*******s 的大作中提到】
: X-rays of the chest and shoulder.
: Dissection?

A*******s
发帖数: 9638
13
Typical case, yes。
Not sure if the pain is associated with breathing.

【在 d****y 的大作中提到】
: The pain should be sharp in dissection.
s*******1
发帖数: 428
14
bacterial endocarditis?
Next step: blood culture, echo.

【在 V*****G 的大作中提到】
: I have a real case, just roughly know the history, please bear with me.
: A 62 yo w m c/o left shoulder pain for several days
: Pain is dull, 6/10,
: Caught a cold several days before onset of shoulder pain
: With fatigue, slight short of breath
: Non smoker
: Past history is HTN x 15 years, on diuretics
: No travel history
: No trauma/injury history
: Occupation: office based job

V*****G
发帖数: 337
15
this guy finally went to the hospital. It turns out this guy had a heart
attack, 3 vessel stenosis, had a CABG
before released from hospital.

【在 V*****G 的大作中提到】
: I have a real case, just roughly know the history, please bear with me.
: A 62 yo w m c/o left shoulder pain for several days
: Pain is dull, 6/10,
: Caught a cold several days before onset of shoulder pain
: With fatigue, slight short of breath
: Non smoker
: Past history is HTN x 15 years, on diuretics
: No travel history
: No trauma/injury history
: Occupation: office based job

A*******s
发帖数: 9638
16
I was looking for zebras.
Yes, always check cardiac enzymes and EKG for the referred pain.

【在 V*****G 的大作中提到】
: this guy finally went to the hospital. It turns out this guy had a heart
: attack, 3 vessel stenosis, had a CABG
: before released from hospital.

A*******s
发帖数: 9638
17
昨天有同仁对我的一个case感兴趣, 我今天查了下病历, 给大家写个完整的。
53 yrs old WM, previously healthy, presented with intractable fatigue and
weight loss for 3 months. A comprehensive workup by his PCP including a CT
of chest was reported as unremarkable.
PMH: Asthma, seeing a pulmonologist who reviewed chest CT and agreed with
radiologist's report.
Soical. Smoker 1 ppd. No ETOH/Drug.
FMH: neg for CA.
ROS: Not remarkable, no diplopia, dysphagia, no fever, no joint pain, no
muscle ache.
PE: Vital normal.
No positive findings on PE.
Lab: CBC/CMP: nl, ESR/CRP: nl; CPK: nl.
CT of Chest: single lymphadenopathy, non-specific.
I have no clue at the first visit. I ordered a ACHR AB to r/o Myasthenia
Gravis. and ampirically started him on Mestinon. 1 week later, he returned
and told me he had improved after being on mestinon. But his ACHR was neg,
as well as MUSK antibody. I started to cast doubt on the diagnosis. So I re
-examed him and again everything was normal except mild weakness on right
hand grip, very, very subtle. So I went ahead for brain CT. Why CT? Because it
was cheap and no preapproval, and I was not that sure.
Surprising there was a tumor in the left frontal lobe. I suddenly understood
what was going on. I called the radiologist who read the chest CT and requested
him to do the lymph node biopsy. The pathology report comfirmed small call
CA. I sent him to a famous academic cancer center.
Interestingly, I was not able to taper off the mestinon. Anti-Hu antibody
title was very high. I started to suspect he has ELS instead of MG.
The academic cancer center sent him to neurology department and they
comfirmed he has ELS, one of the paraneoplastic syndrome. They recommended
him to continue mestinon for ELS, interestingly.

It has been 2.5 yrs already. He has been under serial chemo/radiation
treatements and can not have regular clinic visits due to immunosuppression.
Conclusion:
This patient's fatigue symptom is not from brain metastasis or lung Ca per
se. Instead, ELS caused his clinic symptoms and led to the earlier diagnosis.
V*****G
发帖数: 337
18
Nice, first time to know how ELS presented in the real case. It is just one
thing, that's it, how nice it is! And again, PE is very important and assume
everything is real (not the opposite).

CT

【在 A*******s 的大作中提到】
: 昨天有同仁对我的一个case感兴趣, 我今天查了下病历, 给大家写个完整的。
: 53 yrs old WM, previously healthy, presented with intractable fatigue and
: weight loss for 3 months. A comprehensive workup by his PCP including a CT
: of chest was reported as unremarkable.
: PMH: Asthma, seeing a pulmonologist who reviewed chest CT and agreed with
: radiologist's report.
: Soical. Smoker 1 ppd. No ETOH/Drug.
: FMH: neg for CA.
: ROS: Not remarkable, no diplopia, dysphagia, no fever, no joint pain, no
: muscle ache.

d**o
发帖数: 618
19
Thank you very much for the detailed description. This patient is really
lucky!

【在 A*******s 的大作中提到】
: 昨天有同仁对我的一个case感兴趣, 我今天查了下病历, 给大家写个完整的。
: 53 yrs old WM, previously healthy, presented with intractable fatigue and
: weight loss for 3 months. A comprehensive workup by his PCP including a CT
: of chest was reported as unremarkable.
: PMH: Asthma, seeing a pulmonologist who reviewed chest CT and agreed with
: radiologist's report.
: Soical. Smoker 1 ppd. No ETOH/Drug.
: FMH: neg for CA.
: ROS: Not remarkable, no diplopia, dysphagia, no fever, no joint pain, no
: muscle ache.

A*******s
发帖数: 9638
20
I felt I was flattered. :)

【在 d**o 的大作中提到】
: Thank you very much for the detailed description. This patient is really
: lucky!

相关主题
Funny story about OSH哈医血案
有包子!求macrocytic anemia 的鉴别诊断求教:妈妈冠心病的治疗选择(附CT图)
逆风妹妹的小说哪去啦?冠心病,CT、PCI、CABG......砍掉Cable贴!!
进入Medicalpractice版参与讨论
s********o
发帖数: 3319
21
【 以下文字转载自 MedicalCareer 讨论区 】
发信人: herby (小臭宝), 信区: MedicalCareer
标 题: [合集] 你有生以来做出的最佳临床诊断是什么?
发信站: BBS 未名空间站 (Thu Jun 2 00:01:26 2011, 美东)
☆─────────────────────────────────────☆
Aplusplus (Hakuna Matata) 于 (Sun Mar 27 11:11:49 2011, 美东) 提到:
首先我申明,我放弃我的ignore list, 欢迎所有ID参加。
每个医生护士或者其他相关人员, 无论在中国还是美国, 肯定都有一些case刻骨铭心
。 请大家把你印象最深的一例写出来, 与大家分享。 比方说, 清华男被鉈杀案,
那个中国来的护士就是诊断的关键, 相信本例就是她一辈子也忘不了的诊断。
我觉得这样的讨论有助于大家提高学医热情,丰富医学知识,开拓临床思维, 间接帮
助考版医生复习,准备CS,和 interview. 你要是能从中挑一个case让你PD
impressed,我想我的目的就达到了。
希望所有ID, 包括考生, resident, fellow, attending,PA/NP, nurse 和中国来
的医生不再从事医疗工作的, 都能奉献你的pearl. 英文中文都行。
我先来:
CC: Left leg pain for 5 days.
35 yrs old female woke up with left foot pain. Saw her PCP, had x-rays, and was
given motrin. No improvement but worse with left leg pain as well. See
another PCP again, diagnosis was sciatica. She was given neurontin. No
effect. She came to me on herself. (walk-in)
After my exam, I agreed with her PCP's assessment and management. But before
the patient put on her socks, I noticed the left foot skin color was a
little different from the other side. So I put my hands on both her feet
simutanously: One was warm and the other was "ice" cold.
I ordered a stat vascular U/S.
The patient had an emergent surgery for left leg popliteal artery thrombosis
. She was a heavy smoker.
☆─────────────────────────────────────☆
Aplusplus (Hakuna Matata) 于 (Sun Mar 27 11:17:33 2011, 美东) 提到:
I forgot to attach the picture.
☆─────────────────────────────────────☆
abcd2010 (abc) 于 (Sun Mar 27 11:59:14 2011, 美东) 提到:
您应该也整个专线。
☆─────────────────────────────────────☆
herby (迎春) 于 (Sun Mar 27 12:02:42 2011, 美东) 提到:
Buerger’s Disease?
☆─────────────────────────────────────☆
Aplusplus (Hakuna Matata) 于 (Sun Mar 27 12:19:35 2011, 美东) 提到:
Right,Buerger’s Disease is vasculitis and its symptoms could be close to this case.
I believe she has ischemic PVD due to smoking.
希望case 不单单是最后诊断。
You can present the case as a brief SOAP Note:
1. Subjectives, 病人的症状
2. Objectives, 你的发现, 包括PE, lab, images, etc
3. Assessment, 你的诊断, 为什么。
4. Plan。 治疗意见, 有好的结果最佳!
☆─────────────────────────────────────☆
Aplusplus (Hakuna Matata) 于 (Sun Mar 27 12:31:13 2011, 美东) 提到:
你也来个case吧,不需要很复杂的。
我强调的是过程, 不光是结果。
大家随便说说, 我已经抛砖了。。。
☆─────────────────────────────────────☆
chugol (看风景) 于 (Sun Mar 27 13:34:22 2011, 美东) 提到:
你觉得为什么之前的PCP误诊的原因是什么?
经验不够?还是这个病人的病程决定的?譬如说在这个病人在看你之前她的症状还没有
这么坏,artery thrombosis in its early stage, for instance.
你个人从这个病例中总结出的经验是什么呢?
☆─────────────────────────────────────☆
Aplusplus (Hakuna Matata) 于 (Sun Mar 27 14:00:53 2011, 美东) 提到:
Q: 你觉得为什么之前的PCP误诊的原因是什么?
A: 没有A++的水准? LOL
Q: 经验不够?还是这个病人的病程决定的?
A: 可能两者皆有。 你其实回答了你自己的问题。
Q:你个人从这个病例中总结出的经验是什么呢?
A: 1. 不要相信Telemedicine. 有诱惑, 曾经想在家工作,后来觉得risk太大。
2. 细节一定不要放过, 我想我的诊断来自观察, 愿意蹲下摸她的“臭脚”,LOL
3. 不要想当然take existing diagnosis.
☆─────────────────────────────────────☆
rainingcats (喵喵) 于 (Sun Mar 27 15:10:51 2011, 美东) 提到:
赞!
不过我没什么经验,围观一下
this case.
☆─────────────────────────────────────☆
cqfly (cq) 于 (Sun Mar 27 15:34:38 2011, 美东) 提到:
同赞,排队围观.
☆─────────────────────────────────────☆
chestnuts (chestnut) 于 (Sun Mar 27 15:43:33 2011, 美东) 提到:
year ago.
did not see the patient. History by pt's daughter, my co-worker
53 years old femal, no medical history, c/o both leg weakness. Pt wen to a
clinic getting some Chinese meds IV drip, no improvment. LOL
no PE
postive family history: brother and sister hypothyrodism
suggestion: CMP and thyroid hormone
Feedback: she was diagnosed as hyperthroidsm
no detail, as I said, not my patient.
I was so proud, because this co-worker treated me totally different after
that.
☆─────────────────────────────────────☆
stardust (水晶) 于 (Sun Mar 27 17:55:28 2011, 美东) 提到:
Firstly, thanks LZ for the great thread. I will follow once I have a good
one.
@chestnuts: You are great! did you make the suggestion by their history or
something else?
☆─────────────────────────────────────☆
abcd2010 (abc) 于 (Sun Mar 27 18:19:04 2011, 美东) 提到:
我有生以来给人最伟大的诊断就是: Alzheimer's disease, 但好像被版主给删了。不支持我的诊断。
☆─────────────────────────────────────☆
Aplusplus (Hakuna Matata) 于 (Sun Mar 27 19:17:05 2011, 美东) 提到:
Interesting.
Thyroid disease presentations are so non-specific that the diagnosis is
somewhat difficult. I remember when I was an medical student in China, one
of my classmates made a diagnosis of Grave's disease at his cardiology rotation
after the patient was misdiagnosed as heart disease by multiple senior
residents and attendings. He ended up being an endocrinologist.
☆─────────────────────────────────────☆
chestnuts (chestnut) 于 (Sun Mar 27 19:23:27 2011, 美东) 提到:
Hyperthyroidism can cause myositis, periodic paralysis, low potassium, etc,
either can lead to weakness.
So far I am still not clear the family trend of thyroid disease, but
periodic paralysis, or periodic hypokelamia has family trait.
Got thoes from my Endo rotation, I had a patient who had myostitis secondary
to hyperthyroidism.
☆─────────────────────────────────────☆
SUMO2009 ((MAP)) 于 (Sun Mar 27 19:29:55 2011, 美东) 提到:
One of the students told me exactly the same story when he did his
rotation in our department.
one
rotation
☆─────────────────────────────────────☆
Aplusplus (Hakuna Matata) 于 (Sun Mar 27 20:30:02 2011, 美东) 提到:
Smart people thinks same, lol.
P.S. Dr.Newbie says: Great minds think alike.
☆─────────────────────────────────────☆
Aplusplus (Hakuna Matata) 于 (Sun Mar 27 20:39:51 2011, 美东) 提到:
Alzheimer's diagnosis = great diagnosis. Are you sure?
I know Ronald Reagan started to have symptoms 13 years prior to the
diagnosis. If you were able to call him "an idiot" during his presidency,
which I bet he had been called multiple times, you were probably great. But
that kind of call does not take a doctor's degree to make, lol.
Bring it up, let us enjoy...

不支持我的诊断。
☆─────────────────────────────────────☆
Aplusplus (Hakuna Matata) 于 (Sun Mar 27 20:48:49 2011, 美东) 提到:
That's why we always check TSH for chronic fatigue syndrome.
,
secondary
☆─────────────────────────────────────☆
Aplusplus (Hakuna Matata) 于 (Sun Mar 27 21:01:08 2011, 美东) 提到:
We are waiting...
I know everyone has at least one great case, how about our BZ/BFs?
☆─────────────────────────────────────☆
skyscorpio (天之蝎子) 于 (Sun Mar 27 22:28:04 2011, 美东) 提到:
赞一个!
贵在坚持哈 :)
☆─────────────────────────────────────☆
Aplusplus (Hakuna Matata) 于 (Sun Mar 27 22:32:36 2011, 美东) 提到:
你来一个case?
☆─────────────────────────────────────☆
skyscorpio (天之蝎子) 于 (Sun Mar 27 22:38:00 2011, 美东) 提到:
掩面疾走
☆─────────────────────────────────────☆
Aplusplus (Hakuna Matata) 于 (Sun Mar 27 23:39:15 2011, 美东) 提到:
C'mon, don't go away.
Ok, since you do not want to contribute, please read this:
When your program director interviews you and asks you:“I noticed on your
CV your nickname at mitbbs is Scorpion, would you mind telling me why
chinese people eats Scorpion? "(see picture)
Your answer?
☆─────────────────────────────────────☆
abcd2010 (abc) 于 (Mon Mar 28 00:24:24 2011, 美东) 提到:
照片上的吃客不一定是Chinese。
☆─────────────────────────────────────☆
skyscorpio (天之蝎子) 于 (Mon Mar 28 10:21:23 2011, 美东) 提到:
uPig?
LOL
your
☆─────────────────────────────────────☆
Aplusplus (Hakuna Matata) 于 (Mon Mar 28 12:28:33 2011, 美东) 提到:
Actually the answer is for pain.
Where is Dr.Lexian?
iPig
☆─────────────────────────────────────☆
Lexian (蒙古大夫) 于 (Mon Mar 28 15:10:16 2011, 美东) 提到:
oh yeah? u sure? i thought ppl eat it because it tastes like shrimp. lol
scorpion poison might have some anesthetic effect, but not a bbq one like in
your picture though. hehe
☆─────────────────────────────────────☆
pineyan3 (pineyan3) 于 (Mon Mar 28 15:55:35 2011, 美东) 提到:
I have a sad case.
I was rotating at a US #1/2 cancer center.
A 40 Yr old man came in due to HA. He was diagnosed with Burket's lymphoma
and had chemotherapy several weeks ago. his labs shows leukopenia that is
likely from the chemo. He had CT, MRI that are showed white matter changes
and some sinus wall thickening. I did an LP which showed several lymphocytes
. I asked the neurosurgery to perform a biopsy but they refused. My top 1
differential was Mucor. since i was only a starting second year resident, no
one wants to listen to me, including my attending (who was a good
oncologist), ID atttending. They all think it is lymphoma involvement of the
brain. anyways, the patient lost his left eye sight on the second day of
admission, his right eye sight on thirdday, then paralysed on one side the
fourth day. he died in 1 week. an autopsy showed mucomycosis.
it is one of the reasons that I did not choose oncology as my subspecialty.
I just can't forget the many patients who died during my two months of
oncology rotation. it just makes me sad.
☆─────────────────────────────────────☆
chestnuts (chestnut) 于 (Mon Mar 28 16:12:01 2011, 美东) 提到:
interesting, lyphoma, chemo, lyphoma involvment vs mucor infection, sinus
wall thickening.
just wonder is there other way to test the spinal fluid for mucor infection
, or biopsy is the only way.
lymphocytes
no
the
☆─────────────────────────────────────☆
Aplusplus (Hakuna Matata) 于 (Mon Mar 28 19:40:53 2011, 美东) 提到:
Great case. Pearls in this case include:
1. Immunosuppressed patients are vulnerable to fungus infection. When
headache is present, CNS infection should be considered, besides the
leptomeningeal carcinomatosis.
2. CSF neg does not necessorily R/O CNS infection.
3. CNS Mucor most likely comes from invasion of sinus mucor infection
directly. If that was the case, you might consult ENT instead of NSG. If
sinus mucor can be comfirmed, treat the sinusitis aggressively instead of
encephalitis.
4. Avoid brain biopsy as much as possible if you suspect an infection.
Do you remember how many cells(mono, not lymph in CSF)exactly? What was
protein/glucose? What was india ink? what about fungal culture?
At any rate, given the CNS fungal infection, chance to survive is almost nil
.
lymphocytes
no
the
☆─────────────────────────────────────☆
Aplusplus (Hakuna Matata) 于 (Mon Mar 28 19:53:07 2011, 美东) 提到:
I believe you can do Mucor PCR.
infection
☆─────────────────────────────────────☆
Aplusplus (Hakuna Matata) 于 (Mon Mar 28 20:13:57 2011, 美东) 提到:
I think Scorpion may be good for Rheumatoid Arthritis(RA) which causes
chronic pain. So it is deemed as analgesics.
If Scorpion has ture analgesic effect, the patient may just have a breakfast
of scorpion prior to surgery. No more needs for Dr.Lexian. And you just
need to set up a farm for raising scorpion instead. lol.
I don't think anyone eats scorpion. Just for medical use. Someone exaggerates it.
in
☆─────────────────────────────────────☆
NPtobe (tiantian) 于 (Tue Mar 29 02:29:37 2011, 美东) 提到:
Also make sure check B12 level on old people who coming in with MS changes
or psychosis.
☆─────────────────────────────────────☆
NPtobe (tiantian) 于 (Tue Mar 29 02:30:22 2011, 美东) 提到:
Also make sure check B12 level on old people who coming in with MS changes
or psychosis.
☆─────────────────────────────────────☆
NPtobe (tiantian) 于 (Tue Mar 29 02:49:52 2011, 美东) 提到:
38yr, female, present with diarrhea for 4 days, initially loose watery
diarrhea, 3-4 times/days, now every hour, and blood mixed with the stool,
medium volume, non-foul smelling, with mild crampy abd pain. Had a course of
antibiotics (Keflex) 2 weeks ago
Labs:
LFTs/Amylase/Lipase: WNL
Chemo: Na 137 K 3.4 Cl 108 CO2 23 BUN 6 Cr 0.6 Glu
127
CBC: WBC 13.6 Hgb 12.4 HCT 37.9 Plt 264
Coag: PT 12.2 INR 1.1 PTT 29
Hospital Course:
1. IVF, po hydration
2. Stool Cultures, Stool O&P *3, Stool C.diff *3
3. Abx: Flagyl 500mg po Q8hrs + Levaquin 750mg IV qd
4. CT A/P with contrast was performed, which showed sigmoid colon mild to
moderate wall thickening, possibly representing colitis of infectious
etiology, with inflammatory bowel disease not excluded.
5. GI consulted, for Flex Sigmoidoscopy
☆─────────────────────────────────────☆
Aplusplus (Hakuna Matata) 于 (Tue Mar 29 17:42:28 2011, 美东) 提到:
Would you please finish your case? Please also tell us why this case is so
imressive to you.
Thanks.
of
☆─────────────────────────────────────☆
Mindset (Tough) 于 (Wed Mar 30 00:18:55 2011, 美东) 提到:
House is one of my favorite shows. In one episode, House started one treatment
plan cause all other possibilities are fatal. Was it why other attendings did
take mucor seriously?
My best diagnosis: Lupus.
☆─────────────────────────────────────☆
Aplusplus (Hakuna Matata) 于 (Wed Mar 30 12:49:11 2011, 美东) 提到:
Would you present the case of lupus? I can not wait.....
treatment
attendings did
☆─────────────────────────────────────☆
Aplusplus (Hakuna Matata) 于 (Fri Apr 1 21:26:42 2011, 美东) 提到:
Weekend case:
52 yrs old male, single, lives along, was sent to ER by the girlfriend for
new onset confusion. He was initially hallucinating and become drowsy and
progressively worse over 2-3 days. He just saw his neurologist a day before
and was sent to home, but no details available. He was directly admitted to
ICU.
PMH: ETOH abuse with pancreatitis. But he quits ETOH for 15 years. He has been seeing his neurologist for chronic pancreatits per his girlfriend.
Social: No current smoking or ETOH abuse. Used cocaine 1 month ago, but this
time all drug screen is neg.
PE: T: 100.4 F, BP 140/90, HR: 100, Stuporous,combative, sweating,
periodic myoclonus of all extremities. No focal weakness. Neg Babinski.
CT of Brain is neg. CBC, CMP and NH3 is normal. Drug screen is neg.
LP is neg.
He got Ativan 6 mg total and Haldol 5 mg, no changes.
A dose of Drug A dramatically reversed his symptoms.
BZ for first right answer. BZ/BFs, how much are you going to sponsor? I'll
match...
☆─────────────────────────────────────☆
skyscorpio (天之蝎子) 于 (Fri Apr 1 22:19:53 2011, 美东) 提到:
您老是前辈,您说奖励多少包子我们版务就奖励多少,呵呵
看着不像很难的样子,虽然我自己不知道答案 :)
before
to
this
☆─────────────────────────────────────☆
nanajj (Bumblebee Doc) 于 (Fri Apr 1 22:20:58 2011, 美东) 提到:
Serotonin syndrome - cyproheptadine
before
to
this
☆─────────────────────────────────────☆
Aplusplus (Hakuna Matata) 于 (Fri Apr 1 22:24:46 2011, 美东) 提到:
Haha, 10 wb if within 10 answers.
Why don't you try it? I give u 50 if u hit it.
☆─────────────────────────────────────☆
Aplusplus (Hakuna Matata) 于 (Fri Apr 1 22:27:30 2011, 美东) 提到:
Good thought, but there is no history supporting serotonin syndrome.
This is a real case I managed in ICU.
I just added an additional clue: I called his girlfriend and she claimed he had been seeing his neurologist for chronic pancreatitis, believe or not.

☆─────────────────────────────────────☆
chipmunk (花栗鼠) 于 (Fri Apr 1 22:47:05 2011, 美东) 提到:
From hx and available w/u, delirium tremens, intoxications, infx dz like
encephalitis, meningitis are less likely. Pt saw neurologist and might start
a neuroleptic medication. Therefore, NMS - dopamine agonist?
Thank u!
he had been seeing his neurologist for chronic pancreatitis, believe or not.
☆─────────────────────────────────────☆
Aplusplus (Hakuna Matata) 于 (Fri Apr 1 22:55:08 2011, 美东) 提到:
Sorry, he was not on any neuroleptics.
You may use D2 agonist for NMS, but more useful drug is benzo.
start
not.
☆─────────────────────────────────────☆
lll00000 (00000) 于 (Fri Apr 1 22:59:09 2011, 美东) 提到:
spontaneous bacterial peritonitis?
for
☆─────────────────────────────────────☆
Aplusplus (Hakuna Matata) 于 (Fri Apr 1 23:01:22 2011, 美东) 提到:
May I ask you why? There was no sign of sepsis.
☆─────────────────────────────────────☆
lll00000 (00000) 于 (Fri Apr 1 23:19:57 2011, 美东) 提到:
oops,no PE,no sepsis.
alcohol withdrawal? drug A is benzodiazepine?
☆─────────────────────────────────────☆
Aplusplus (Hakuna Matata) 于 (Fri Apr 1 23:23:51 2011, 美东) 提到:
Nope.
☆─────────────────────────────────────☆
skyscorpio (天之蝎子) 于 (Fri Apr 1 23:25:17 2011, 美东) 提到:
慢性胰腺炎去看神经科,有意思
☆─────────────────────────────────────☆
Aplusplus (Hakuna Matata) 于 (Fri Apr 1 23:32:28 2011, 美东) 提到:
haha, you almost get it!
☆─────────────────────────────────────☆
Jolie2010 (dreamseeker) 于 (Fri Apr 1 23:49:49 2011, 美东) 提到:
what the blood sugar level?
☆─────────────────────────────────────☆
herby (迎春) 于 (Sat Apr 2 00:10:43 2011, 美东) 提到:
偶去google了一下,瞎说几句: 慢性胰腺炎的神经痛?然后用了opioid,然后出现了
withdrawal syndrome?
☆─────────────────────────────────────☆
carrie66 (麦地听风) 于 (Sat Apr 2 00:16:50 2011, 美东) 提到:
Morphine to control Chronic Pancreatitis Neuropathic Pain--Naloxone
for
and
before
admitted to
has been seeing his neurologist for chronic pancreatits per his
girlfriend.
but this
☆─────────────────────────────────────☆
Aplusplus (Hakuna Matata) 于 (Sat Apr 2 08:29:52 2011, 美东) 提到:
Bingo!
This guy has been seeing his neurologist for chronic pain. He got his
narcotics prescription that day but did not get refill right away. Then he
developped withdrawal syndrome and became confused.
A dose of Morphine reversed his symptoms in 5 min. I started him on oral
opioids and he was discharged in 2 days.
I have to go now and great job for Herby.
☆─────────────────────────────────────☆
Aplusplus (Hakuna Matata) 于 (Sat Apr 2 08:32:55 2011, 美东) 提到:
U are very close.....
☆─────────────────────────────────────☆
herby (迎春) 于 (Sat Apr 2 10:59:29 2011, 美东) 提到:
多谢前辈,很有意思的一个case :)
he
☆─────────────────────────────────────☆
skyscorpio (天之蝎子) 于 (Sat Apr 2 11:51:17 2011, 美东) 提到:
这个例子不错!结论就是要珍惜生命,远离烟酒 :)
我转了100伪币请您代发吧
he
☆─────────────────────────────────────☆
Lexian (蒙古大夫) 于 (Sat Apr 2 14:34:40 2011, 美东) 提到:
good case. how u suggest to manage him after this fiasco?
☆─────────────────────────────────────☆
Aplusplus (Hakuna Matata) 于 (Sun Apr 3 00:05:54 2011, 美东) 提到:
Thanks.
I think the key for this case is getting a good history. If you get a good
history, you make a good diagnosis.
I posted this case 2 yrs ago in a physician web site. Nobody got right
answer: Call the girlfriend for more history.
Well, Kudos to Herby BF. Good job. That is why I always think, CMGs can do
as well as AMGs, if not even better.
蝎子BZ was right in the front of door, but he seemed to be too shy to knock
it, didn't he? lol.
I'll use up this 100 wb, many thanks to BZ.
☆─────────────────────────────────────☆
Aplusplus (Hakuna Matata) 于 (Sun Apr 3 00:06:56 2011, 美东) 提到:
I sent him to Dr.Lexian for pain management.
☆─────────────────────────────────────☆
NPtobe (tiantian) 于 (Sun Apr 3 00:24:42 2011, 美东) 提到:
38yr, female, present to PAH on 4/4/10 with diarrhea for 4 days, initially
loose watery diarrhea, 3-4 times/days, now worse and loose watery stool
every hour, and blood mixed with the stool, medium volume, non-foul smelling
, with mild crampy abd pain. Have had 5 BMs since yesterday, and the last
one almost all blood. (+) subjective fever, no chills, no cough/SOB. Had a
course of antibiotics (Keflex) 2 weeks ago (3/9/10- 3/14/10).
PMHx: Kidney stones, Seizure disorders, Vit D deficiency
PSHx: Sinus Surgery 3/10/10 for deviated nasal septum
FHx: NC
SHx: (-) smoking/ ETOH/IVDA, mental health worker
Allergies: Codeine
ROS: all negative, except as noted in HPI
PE:
Appearance: NAD
HEENT: no sclera icterus, PERRLA
PULM: B/L lungs CTAs,
CV: (+) S1, (+)S2, RRR, (-) murmurs
GI: Abd soft, mild tenderness, nondistended, (+) BS in 4 quadrant
NEURO: AAO*3
Labs:
LFTs/Amylase/Lipase: WNL
Chemo: Na 137 K 3.4 Cl 108 CO2 23 BUN 6 Cr 0.6 Glu
127
CBC: WBC 13.6 Hgb 12.4 HCT 37.9 Plt 264
Coag: PT 12.2 INR 1.1 PTT 29
Hospital Course:
Pt was admitted on 4/4/10
1. IVF, po hydration
2. Stool Cultures, Stool O&P *3, Stool C.diff *3
3. Abx: Flagyl 500mg po Q8hrs + Levaquin 750mg IV qd
4. CT A/P with contrast was performed, which showed sigmoid colon mild to
moderate wall thickening, possibly representing colitis of infectious
etiology, with inflammatory bowel disease not excluded.
5. GI consulted, for Flex Sigmoidoscopy on 4/5/10
4/5/10 AM:
Stool cultures: (-) Salmonella/Shigella/Campylobacter/Giardia/Crytosporidium
Antigen isolated in 24 hours
Flex Sigmoidoscopy: Diffuse moderate inflammation characterized by erosion,
erythema, friability, and shallow ulcerations was found in the sigmoid colon
and in the descending colon with rectal sparing, likely secondary to Crohn
’s disease. Biospies was taken with cold forceps for histology.
Dx: Crohn’s disease
Recommendations: Prednisone 40mg po QD
Outpt GI F/U in 10-14 days
4/6/10:
C.diff (-) *1
Tolerated clear liquid diet, started on low residue diet
Biopsy: mild acute colitis with crypt abscesses, no granulomata are seen, no
chronic architectural changes identified
Pt D/C’ed on prednisone 40mg po QD, F/U with GI as outpt in 10-14 days,
will consider immunomodulator or biologic agent as outpt
4/7/10:
Lab called stool culture came back: (+) shigella
Called patient at home, told her to taper steroid, and start cipro 500mg po
bid*5 days, pt is thrilled.
Requestion the patient about sick contact:
Pt is Jewish, and during Passover, had dinner with family members from upper
New York, some of them was sick.
Lesson:
Always make sure for negative stool studies before start steroid.
☆─────────────────────────────────────☆
Aplusplus (Hakuna Matata) 于 (Sun Apr 3 00:51:25 2011, 美东) 提到:
Good case.
I think the management is reasonable.
First, based on initial neg culture and positive biopsy, steroids is the
right treatment.
3 days later, shigella became positive, I wonder why not just stop steroids?
I believe in this case a tapering dose of steroids is still the right
treatment. Just like we commonly use medrol dosepack.
Again, the diagnosis would be simple if the intern got more history from the
family, LOL.
smelling
a
☆─────────────────────────────────────☆
chestnuts (chestnut) 于 (Sun Apr 3 10:26:03 2011, 美东) 提到:
I would like to present another case that is not diagnosed by me.
I took care of this 17 years old girl 2 days ago, who came to surgical floor
for VP shunt replacement for her pseudo cerebri disease. Primarily she
presented to neurologist for her 7 years history of headache, which is
constant, both sides, 4-5/10 in intensity, vague trigger factor, acerbating
two times every month,lasting one week, at the point, pain would reach 8-9/
10 in intensity.
FH: her older sister has psychiatry diagnosis. Parents are healthy, some
secondary realtives postive with autoimmunive disease.
PE: develop well, slightly malnourished .negative for other signs.
lab: Brain ct,MRI, EEG negative. Anti ssa ssb positive (for which she is
following the rheumatologist)
LP tap three times , open pressure as 20, 24. 40
pseudo cerebri is diagnosed , pt strongly requests VP shunt surgery,
immediately after surgery , the headache is gone.
The patient has been seeking for the dx and treatment for sever years, she
was diagnosed as migraine, seizure, etc.Finally they came to this
neurologist who specializes in headache.
I myself had bad headache yesterday for the whole day, I am sincerely glad
for this girl.
☆─────────────────────────────────────☆
Aplusplus (Hakuna Matata) 于 (Sun Apr 3 12:00:57 2011, 美东) 提到:
Is this girl obese?
Young female, over weight, with chronic daily headache, should be considered as
pseudotumor cerebri. On exam, you may find papillary edema. Treatment
includes diamox and LP. VP shunt is the final step. It is a brain surgery.
floor
acerbating
☆─────────────────────────────────────☆
chestnuts (chestnut) 于 (Sun Apr 3 18:06:14 2011, 美东) 提到:
She is very very slim, onset age is 10 years old. Not quite typical though.
considered as
surgery.
☆─────────────────────────────────────☆
Aplusplus (Hakuna Matata) 于 (Wed Apr 13 20:22:42 2011, 美东) 提到:
Just see it.
This reminds me a similar case: A very slim girl about 12 yrs old has
pseudotumor.
☆─────────────────────────────────────☆
Aplusplus (Hakuna Matata) 于 (Wed Apr 13 20:51:25 2011, 美东) 提到:
This case is tough. 20 WB for the first. (from BZ)
A 35 yrs old WF, was sent to ER by her husband in AM since he could not wake
her up.
The patient was entirely normal before last dinner. They planned to have
BBQ, but after she drank a can of beer and 3 cans of soda, she felt sick. So
she went to bed directly.
At ER, she was stuporous, a drug screen was neg. CT of brain, EKG, CXR were
all normal. LP is neg. UA(-), Blood culture was sent. All lab was normal
except Glucose was 60. So she was given D5 1/2NS and tranferred to ICU.
When she arrived ICU, she was comatose and subsequetly she was intubated.
PMH. Healthy except Gastric bypass surgery 6 month ago.
Social: No Cig/ETOH abuse, no drug abuse.
FMH: CAD, HTN.
ROS: N/A
PE: BP: 90/60, P: 45-55, Temp: 93.5 F
Gen: Comatose, on vent.
HEENT: Normal.
Neck: supple.
Chest/CVS: Normal except bradycardia.
Abdomen: Normal.
GU: Normal
Skin: No bruises/ Rash.
Neuro: PERRL. (+)Doll's eye.
(+)pain withdrawal bilaterally. Low muscle tone. No jerking.
(-)Babinski.
Workup in ER please refer to history.
A treatment rapidly reversed her sysptoms.
What is your assessment and plan?

☆─────────────────────────────────────☆
yuzaiusa (Expecto Patronum!) 于 (Wed Apr 13 22:05:54 2011, 美东) 提到:
alcohol intoxication?
treated with HD?
☆─────────────────────────────────────☆
Aplusplus (Hakuna Matata) 于 (Wed Apr 13 22:17:04 2011, 美东) 提到:
No, please read the info carefully:
1. She is not alcoholic.
2. Drug screen is negative.
☆─────────────────────────────────────☆
Lexian (蒙古大夫) 于 (Wed Apr 13 22:20:16 2011, 美东) 提到:
Dumping?
☆─────────────────────────────────────☆
daisyy (Daisy) 于 (Wed Apr 13 22:21:50 2011, 美东) 提到:
except Glucose was 60.
Healthy except Gastric bypass surgery 6 month ago.
恩?这病人在减肥?
Chest/CVS: Normal except bradycardia.
难道是beta blocker过量?
Glucagon
☆─────────────────────────────────────☆
snowfox01 (雪山飞狐) 于 (Wed Apr 13 22:27:49 2011, 美东) 提到:
Let me guess,
Dr. Lexian(前辈) almost get the reason. patient who had gastric bypass surgery
will get dumping symptoms, that is why she went to bed within 30~40 min
after take the alcohol. Also after bypass surgery, patient will absorb
alcohol quickly and excrete slowly.
☆─────────────────────────────────────☆
Aplusplus (Hakuna Matata) 于 (Wed Apr 13 22:35:07 2011, 美东) 提到:
You caught a clue.
☆─────────────────────────────────────☆
Aplusplus (Hakuna Matata) 于 (Wed Apr 13 22:36:03 2011, 美东) 提到:
Glucose 60 is borderline.
She is not on any drugs.
☆─────────────────────────────────────☆
snowfox01 (雪山飞狐) 于 (Wed Apr 13 22:37:26 2011, 美东) 提到:
I will go with Yuzaiusa, My guess is: alcohol intoxication and hypoglycemic
shock.
Don't know how to treat the patient.
☆─────────────────────────────────────☆
Aplusplus (Hakuna Matata) 于 (Wed Apr 13 22:39:49 2011, 美东) 提到:
Why hypothermia?
Why Bradycardia?
☆─────────────────────────────────────☆
Aplusplus (Hakuna Matata) 于 (Wed Apr 13 22:41:25 2011, 美东) 提到:
The fact was that she got worse after she received D5 1/2NS.
And I repeat: Drug screen is neg for ETOH. So no intoxication.
hypoglycemic
☆─────────────────────────────────────☆
yuzaiusa (Expecto Patronum!) 于 (Wed Apr 13 22:43:21 2011, 美东) 提到:
What was her ABG?
★ Sent from iPhone App: iReader Mitbbs 6.88 - iPad Lite
☆─────────────────────────────────────☆
Aplusplus (Hakuna Matata) 于 (Wed Apr 13 22:49:56 2011, 美东) 提到:
Normal.
☆─────────────────────────────────────☆
snowfox01 (雪山飞狐) 于 (Wed Apr 13 23:00:01 2011, 美东) 提到:
It must have something to do with the bypass surgery. I know post bypass
surgery can cause B12 deficiency and iron deficiency, not sure about B1
deficiency. May be caused by B1 deficiency and D5 can worsen the symptoms.
☆─────────────────────────────────────☆
daisyy (Daisy) 于 (Wed Apr 13 23:05:03 2011, 美东) 提到:
恩。这个beta blocker虽然使心率减慢,但是降低insulin的释放,是会使血糖升高的
,所以也不合
理。
☆─────────────────────────────────────☆
daisyy (Daisy) 于 (Wed Apr 13 23:09:46 2011, 美东) 提到:
dumping syndrome 糖类进入肠道过快,引起insulin释放过多。 补糖水的话insulin就
继续增
高?
octreotide 可以降低insulin的释放?
☆─────────────────────────────────────☆
Aplusplus (Hakuna Matata) 于 (Wed Apr 13 23:10:38 2011, 美东) 提到:
Why hypothermia?You never think about sepsis?
☆─────────────────────────────────────☆
Lexian (蒙古大夫) 于 (Wed Apr 13 23:12:54 2011, 美东) 提到:
快抢碘盐。
☆─────────────────────────────────────☆
Aplusplus (Hakuna Matata) 于 (Wed Apr 13 23:14:12 2011, 美东) 提到:
Her Glucose had been normal since D5 was given.
☆─────────────────────────────────────☆
Aplusplus (Hakuna Matata) 于 (Wed Apr 13 23:14:52 2011, 美东) 提到:
LOL!
☆─────────────────────────────────────☆
snowfox01 (雪山飞狐) 于 (Wed Apr 13 23:15:08 2011, 美东) 提到:
Blind loop syndrome?
☆─────────────────────────────────────☆
Aplusplus (Hakuna Matata) 于 (Wed Apr 13 23:16:13 2011, 美东) 提到:
Good thought, but why?
Can you explain why bradycardia?
☆─────────────────────────────────────☆
snowfox01 (雪山飞狐) 于 (Wed Apr 13 23:26:26 2011, 美东) 提到:
Since you mentioned sepsis, blind loop syndrome can cause bacteria over grow.
But I still vote for my B1 deficiency caused by post bypass and alcohol and
D5 consumption.
☆─────────────────────────────────────☆
Aplusplus (Hakuna Matata) 于 (Wed Apr 13 23:31:07 2011, 美东) 提到:
So what do you call this disease? And what do you do?
grow.
and
☆─────────────────────────────────────☆
daisyy (Daisy) 于 (Wed Apr 13 23:32:44 2011, 美东) 提到:
如果是sepsis的话那不是要用antibiotics?
☆─────────────────────────────────────☆
Aplusplus (Hakuna Matata) 于 (Wed Apr 13 23:36:10 2011, 美东) 提到:
You are right. She was treated with antibiotics. Sepsis can present as
hypothermia.
☆─────────────────────────────────────☆
snowfox01 (雪山飞狐) 于 (Wed Apr 13 23:41:02 2011, 美东) 提到:
Thanks for the case.
☆─────────────────────────────────────☆
daisyy (Daisy) 于 (Wed Apr 13 23:48:00 2011, 美东) 提到:
她这是很严重的sepsis了。都快septic shock了吧。 hypotention(borderline
though), hypothermia,
bradycardia, 再不及时可能DIC都出来了。
请教前辈象她这种开始没有发热进展迅速的sepsis多见吗? 有个问题,给糖盐水应该
也没有加重她的
病情,只是病情没有得到控制发展的吧?
☆─────────────────────────────────────☆
daisyy (Daisy) 于 (Wed Apr 13 23:52:39 2011, 美东) 提到:
A very good case. Thanks so much!
☆─────────────────────────────────────☆
Aplusplus (Hakuna Matata) 于 (Wed Apr 13 23:55:25 2011, 美东) 提到:
You give up?
The final diagnosis is : Wernicke's Encephalopathy.
Treatment: Thiamine 100 mg iv stat.
Patient dramatically improved after the injection. Her Vitamin B1 level
came back 1 wk later and proved the diagnosis.
Why bradycardia and hypotension, hypothermia? Hypothalamas involvement. But
you need to memorize mammillary body first.
Risk factors? Gastric bypass, poor nutrition, Beer consumption, and giving
D5 without thiamine!
This is not sepsis. Blood culture was negative.
I think Snowfox01 deserves the prize.
Good night.
☆─────────────────────────────────────☆
skyscorpio (天之蝎子) 于 (Wed Apr 13 23:58:39 2011, 美东) 提到:
谢谢!
我其实一眼就想到了 LOL 包子很快发出
But
giving
☆─────────────────────────────────────☆
daisyy (Daisy) 于 (Thu Apr 14 00:01:50 2011, 美东) 提到:
Haha. This makes sense.
I was fooled by your comments.
But
giving
☆─────────────────────────────────────☆
snowfox01 (雪山飞狐) 于 (Thu Apr 14 00:02:32 2011, 美东) 提到:
谢,前辈。
哎,我总赶不上最后一步。“当机遇像雨点般向我洒来,我都镇定自若的一一闪过”
But
giving
☆─────────────────────────────────────☆
snowfox01 (雪山飞狐) 于 (Thu Apr 14 00:03:49 2011, 美东) 提到:
谢, 包子。
☆─────────────────────────────────────☆
Aplusplus (Hakuna Matata) 于 (Thu Apr 14 00:11:35 2011, 美东) 提到:
我早就看出来了, 知道答案的都潜水!
BZ我发。
☆─────────────────────────────────────☆
Aplusplus (Hakuna Matata) 于 (Thu Apr 14 00:13:50 2011, 美东) 提到:
你跟蝎子一样, 美女半夜敲门,你们在门后加锁。
☆─────────────────────────────────────☆
skyscorpio (天之蝎子) 于 (Thu Apr 14 00:14:09 2011, 美东) 提到:
呵呵,您有时间再出一个吧,下回我跳出来献献丑。
☆─────────────────────────────────────☆
Aplusplus (Hakuna Matata) 于 (Thu Apr 14 00:15:02 2011, 美东) 提到:
跟着感觉走。
☆─────────────────────────────────────☆
daisyy (Daisy) 于 (Thu Apr 14 10:32:14 2011, 美东) 提到:
From Wiki:
Thiamine is released by the action of phosphatase and pyrophosphatase in
the upper small intestine. At low concentrations, the process is carrier
mediated and at higher concentrations, absorption occurs via passive
diffusion. Active transport is greatest in the jejunum and ileum (it is
inhibited by alcohol consumption and by folic deficiency).
symptoms.
☆─────────────────────────────────────☆
susufairy (susufairy) 于 (Thu Apr 14 17:16:19 2011, 美东) 提到:
来说个比较特别的例子,谁都没诊断出来而且病人死了。当时在国内一医院当实习医生
见到的,所以不好用英文,一孕39周产妇入院,入院时已开4指,并伴有宫缩疼(注意
,很可能不一定是宫缩疼,我后面会说),入院后生命体征一切正常除了腹疼,偶伴有
恶心,未吐。所有人都认为是宫缩疼,因为开指非常缓慢,记不清多长时间,最终产妇
于第二天行C-SECTION,手术也还顺利,只是手术过程中发现病人血液呈紫色,腹腔中
有小量血性积液。术后两天病人仍腹痛,且有低热,当时考虑是可能术后感染和术后疼,因为是实习医生,再后来做了什么检查就不记得了,后病人诊断为妊辰合并急性坏死性胰腺炎
,病人于C-SECTION后5天死亡,可怜那只有几天的宝宝就这样失去了妈妈。
☆─────────────────────────────────────☆
Aplusplus (Hakuna Matata) 于 (Thu Apr 14 17:41:06 2011, 美东) 提到:
Sad story.
That's why physician should never be dogmatic. Pelvic pain and abdominal
pain are different but if a physician never had the idea that acute
pancreatitis can be a complication of pregancy, he/she just would be
clueless like the case.
Thanks, susufairy, for your contribution. BZ for you. (contributed by bZ
and Snowfox)
Reasons for Acute Pancreatitis and Pregnancy
Most cases of acute pancreatitis in pregnancy are caused by gallstone
disease. It is thought with the weight and hormonal changes induced by
pregnancy, gallstones are more likely to form and thus travel down the
common bile duct to obstruct the pancreas duct outflow. Another proposed
mechanism for acute pancreatitis in pregnancy is high fat levels in the
blood called triglycerides. Again, the hormonal changes of pregnancy can
predispose certain women to developing this condition. When the
triglyceride levels become too high, oxygen cannot adequately travel to the
pancreas via the bloodstream, and pancreatitis can ensue. Of course, all of
the other reasons for developing acute pancreatitis – alcohol use,
reaction to certain medications, trauma to the pancreatic duct – can also
lead to acute pancreatitis in pregnancy
疼,因为是实习医生,再后来做了什么检查就不记得了,后病人诊断为妊辰合并急性坏
死性胰腺炎
☆─────────────────────────────────────☆
yuzaiusa (Expecto Patronum!) 于 (Thu Apr 14 18:56:47 2011, 美东) 提到:
interesting case. I thought EtOH intoxication can suppress the resp. center
, cause hypoxia, then bradycardia. metabolic acidosis can cause bradycardia
and hypotension too. Never thought about vitamin deficiency. Thanks.
But
giving
☆─────────────────────────────────────☆
Aplusplus (Hakuna Matata) 于 (Thu Apr 14 20:06:43 2011, 美东) 提到:
This tough case was mishandled at multiple steps.
At ER, she was found to be hypoglycemic which is very common to cause MS
changes, so she got the D5 without thiamine which made her worse.
Second, she developped hypothermia which led to a diagnosis of Sepsis.
Bradycardia and hypotension were thought to be septic shock.
Fortunately, I took over the case and made the diagnosis after I had a long
talk with her husband.
She hardly had any residual symptoms at the discharge. But if untreated, she
would end up with Korsakoff's syndrome if she survives.
center
bradycardia
☆─────────────────────────────────────☆
Lexian (蒙古大夫) 于 (Thu Apr 14 21:08:05 2011, 美东) 提到:
A big thank you, A , i think everyone joined this discussion wont forget
Wernicke's encephalopathy from now on.
had u seen one in training before this case? I only read about in textbook,
always thought it's a chronic presentation.
This tough case was mishandled at multiple steps.At ER, she was found to be
hypoglycemic........
★ Sent from iPhone App: iReader Mitbbs 6.88 - iPhone Lite
☆─────────────────────────────────────☆
Aplusplus (Hakuna Matata) 于 (Thu Apr 14 21:18:42 2011, 美东) 提到:
Be honest with you, I did not.
But I know, chances favor prepared mind, lol.
,
be
☆─────────────────────────────────────☆
Lexian (蒙古大夫) 于 (Thu Apr 14 21:25:54 2011, 美东) 提到:
I'm sure your internal reward from this case far outweight your 1500 bill,
lol. Hats off to u.
Be honest with you, I did not. But I know, chances favor prepared mind, lol.
★ Sent from iPhone App: iReader Mitbbs 6.88 - iPhone Lite
☆─────────────────────────────────────☆
Aplusplus (Hakuna Matata) 于 (Thu Apr 14 22:25:24 2011, 美东) 提到:
你来个麻醉case? 讲讲张春桥妹妹被laughing gas谋杀之事?
lol.
☆─────────────────────────────────────☆
Lexian (蒙古大夫) 于 (Thu Apr 14 23:28:14 2011, 美东) 提到:
我做的主要是操作, 诊断大多很简单, 就是要on alert, 出状况时反应要快, 处理
要及时。
几年前在一个inner city的trauma center值班,急诊来了个举枪自杀的小伙子, 据说
为情所困, 大概太激愤了的缘故,开枪时头后仰的厉害。 子弹从下巴射入, 居然从
右眼出来。 大脑没事。 送来时半边脸肿成猪头, 嘴里从咽喉以上一片血糊。 ems插
管插不进去。 我操了根纤维镜顺着有气泡的方向往里送, 居然找到气管里。 管插好
了, 其它很快就稳定下来了。 这大概是internal reward最大的case了。呵呵。
☆─────────────────────────────────────☆
Aplusplus (Hakuna Matata) 于 (Thu Apr 14 23:43:31 2011, 美东) 提到:
我最最佩服的是你们插管和腰穿的技巧。仰视。。。
☆─────────────────────────────────────☆
blulue (小黑) 于 (Fri Apr 15 02:16:32 2011, 美东) 提到:
But
giving
相当好的帖子阿,一直跟着看,学到很多东西!!
看了解释,我有几个没想明白的点,自己搜了搜答案,还是有些地方不太明白,想向
Aplusplus前辈请教:
"Wernicke-Korsakoff syndrome results from severe acute deficiency
superimposed on chronic deficiency."
1,Wernicke的表现?考试时候背的classic triad of oculomotor abnormalities,
ataxia, and confusion?
原来经典三联征齐全的,只有20%的病人.这个病人影响到hypothalamas,低压低温.
还有病人影响到thalamas,progressing hearing loss.
2,Chronic deficiency何来?gastric bypass surgery 6 months ago.
B/C Bypass partial small intestine--defect absorption. In early post op
period,difficult to eat enough for enough VitB1.
Google到两个bypass sugery以后VitB1缺乏致病的,一个术后5个月,一个术后4个月.
3,severe acute deficiency 何来?
是beer consumption吗?她没饮酒史,alcoholism造成chronic deficiency算不上.还是
ER补糖不补B1?感觉更像后者...那beer在这起来神马作用呢?
抱歉写得罗里罗嗦,先谢谢前辈!
☆─────────────────────────────────────☆
Aplusplus (Hakuna Matata) 于 (Fri Apr 15 10:36:03 2011, 美东) 提到:
1,Wernicke的表现?考试时候背的classic triad of oculomotor abnormalities,
ataxia, and confusion?
原来经典三联征齐全的,只有20%的病人.这个病人影响到hypothalamas,低压低温.
还有病人影响到thalamas,progressing hearing loss.
-----------------------------------------
Should be hypothalamus, my bad.
This patient was in coma, so ocular symptoms and gait could not be assessed
accurately.
2,Chronic deficiency何来?gastric bypass surgery 6 months ago.
B/C Bypass partial small intestine--defect absorption. In early post op
period,difficult to eat enough for enough VitB1.
Google到两个bypass sugery以后VitB1缺乏致病的,一个术后5个月,一个术后4个月.
-----------------------------------------
Social history is hard to get and never to be reliable. This patient has
history of bypass, and she does drink Beers.
3,severe acute deficiency 何来?
是beer consumption吗?她没饮酒史,alcoholism造成chronic deficiency算不上.还是
ER补糖不补B1?感觉更像后者...那beer在这起来神马作用呢?
-------------------------
Once again, she drinks beers, that's enough. All drinkers are liars,
believe or not. But I can not say she is alcoholic if she or her husband
denies.
ETOH is a big problem for the society.
Thanks for the interest.
☆─────────────────────────────────────☆
gg2 (Allison) 于 (Fri Apr 15 11:28:25 2011, 美东) 提到:
thanks! learned a lot from these case discussions. please keep it on.
☆─────────────────────────────────────☆
blulue (小黑) 于 (Fri Apr 15 12:35:38 2011, 美东) 提到:
assessed
多谢多谢前辈!!
书本跟临床,结合起来学,收获才更大啊!
☆─────────────────────────────────────☆
REDpersimmon (大尾巴睡猫) 于 (Fri Apr 15 15:52:43 2011, 美东) 提到:
大赞此帖!多谢前辈,您让我头一次感受到原来internal medicine也可以这么有趣!
☆─────────────────────────────────────☆
Aplusplus (Hakuna Matata) 于 (Fri Apr 15 16:17:50 2011, 美东) 提到:
Thanks all above for my internal rewards, LOL.
☆─────────────────────────────────────☆
skyscorpio (天之蝎子) 于 (Fri Apr 15 16:28:53 2011, 美东) 提到:
you are great! much better than Dr. Lexain LOL
☆─────────────────────────────────────☆
Aplusplus (Hakuna Matata) 于 (Fri Apr 15 16:42:21 2011, 美东) 提到:
I feel chills. LOL
☆─────────────────────────────────────☆
Lexian (蒙古大夫) 于 (Fri Apr 15 17:10:35 2011, 美东) 提到:
I got the internal reward of keeping A+ here, which is not less than anything
else.
☆─────────────────────────────────────☆
RBC120 (LTC) 于 (Fri Apr 15 20:46:51 2011, 美东) 提到:
在产前检查门诊实习,第一次用听筒在两个不同部位听到胎心,确诊为双胞胎!孕妇高
兴,我也很有成就感。
☆─────────────────────────────────────☆
VictorG (VictorG) 于 (Sat Apr 16 09:12:39 2011, 美东) 提到:
Good case, progressing so fast.
lymphocytes
no
the
☆─────────────────────────────────────☆
VictorG (VictorG) 于 (Sat Apr 16 09:21:24 2011, 美东) 提到:
Narcotics withdrawal----confusion, interesting.
he
☆─────────────────────────────────────☆
VictorG (VictorG) 于 (Sat Apr 16 09:52:23 2011, 美东) 提到:
鹅买高的。
But
giving
☆─────────────────────────────────────☆
VictorG (VictorG) 于 (Sat Apr 16 10:13:40 2011, 美东) 提到:
看了大家的,收获颇丰,也贡献一个,如果算的话。
我有一个CASE, 需要诊断。
一个朋友,N年前买了炸鸡腿,早餐吃了两口,味有点怪,一点都不香,就不吃了。3个
小时以后恶心,呕吐,很快出现眩晕(VERTIGO)很严重的眩晕,无法站立行走,持续
恶心呕吐眩晕,症状持续6个小时左右缓解。客观原因(just started new job)没去医
院(后来朋友后怕,食物中毒,应该去).
因为最后诊断不明确是何种食物中毒造成这么严重的眩晕,希望大家看看。
☆─────────────────────────────────────☆
Aplusplus (Hakuna Matata) 于 (Sat Apr 16 11:49:21 2011, 美东) 提到:
Wallenburg's syndrome.
Your friend had a TIA or stroke in the brain stem. I guess he is a middle
age or older man and has stroke risk factors. Otherwise, I could be wrong.
大家如果觉得我说得有道理, 就复习一下Wallenburg's syndrome吧。 我在医学院神
经科实习时, 就记得这个case, 其他都忘了, lol。
“一个朋友,N年前买了炸鸡腿,早餐吃了两口,味有点怪,一点都不香,就不吃了”
Taste changes
"3个小时以后恶心,呕吐,很快出现眩晕(VERTIGO)很严重的眩晕,无法站立行走,持续
Symptoms exacerbated and involved more area of medulla.
"症状持续6个小时左右缓解"
Possible TIA only.
☆─────────────────────────────────────☆
VictorG (VictorG) 于 (Sat Apr 16 12:13:47 2011, 美东) 提到:
如果是中老年,有危险因素,我想这个肯定top on differential.
那一次发生时朋友很年轻(<30 years old),之后多年也没有过任何illness, 食物中
毒应该是准确的,只是拿不准是那种中毒。
我查了一下,肉毒毒素(clostridium toxin)有可能,可导致vertigo,好像当时也有
weakness, 忘了告诉大家了。
如果是的话,还是很危险的 nausea, vomitting, vertigo, muscle weakness.
,持续
☆─────────────────────────────────────☆
VictorG (VictorG) 于 (Sat Apr 16 12:23:56 2011, 美东) 提到:
再一次说明病例描述病史很重要,忘了病例的年龄,伴发症状,有无其他危险因素以及
其他情况等等。没有这些,看来呈现的完全是另外一种情况。
这个病例,发病是<30 years old, 平素健康,无任何疾病。之后多年也无任何疾病。
☆─────────────────────────────────────☆
daisyy (Daisy) 于 (Sat Apr 16 18:36:10 2011, 美东) 提到:
With such short onset time (3 hours after the food intake), I think it's
most likely caused by preformed toxins produced by bacteria. The
symptoms were gone after 6 hours also support food poison by preformed
toxins.
The most common one is Staphylococcus aureus. The other one is Bacillus
cereus. BC usually be found in reheated rice, so it's unlikely the
culprit in this case.
S.Aureus can be found in meat, mayonnaise, custard. So I think S. Aureus
preformed toxin may be the most likely one.
Pathogenesis: Enterotoxin acts on receptors in gut that transmit
impulses to medullary centers.
Nausea, vomiting, abdominal pain are common symptoms in S. Aureus food
poison. Although vertigo is not a common one, but I think the overall
profile of the case still fits S.Aureus better.
☆─────────────────────────────────────☆
daisyy (Daisy) 于 (Sat Apr 16 18:47:48 2011, 美东) 提到:
Clostridium Botulinum toxin was found in Canned foods (eg, smoked fish,
mushrooms, vegetables, honey).
Presentation:
Descending weakness and paralysis start 1-4 days after ingestion, followed
by constipation.
Mortality is very high.
http://emedicine.medscape.com/article/175569-clinical#a0218
☆─────────────────────────────────────☆
Aplusplus (Hakuna Matata) 于 (Sat Apr 16 22:31:47 2011, 美东) 提到:
Very well summarized, impressive!
Botulinum toxin causes descending paralysis started with diplopia,
dysartheria or dysphagia and facial paralysis. Vertigo is rare.
I agree with you on bacteria toxin. More common symptoms are N/V ,abdominal
pain and diarrhea. Dizziness is common but not vertigo.
When patient says dizzy, make sure if patient means lightheadedness or
vertigo. Big difference.
☆─────────────────────────────────────☆
daisyy (Daisy) 于 (Sat Apr 16 23:29:30 2011, 美东) 提到:
Totally agree with you!
Dizziness is common but not vertigo.
☆─────────────────────────────────────☆
rainingcats (喵喵) 于 (Sun Apr 17 01:59:23 2011, 美东) 提到:
我有过同样的经历啊,朋友送的一盒蛋糕,吃了一个多小时以后恶心,眩晕,站不住,
眼前一黑差点昏过去,吐了才好。
☆─────────────────────────────────────☆
Aplusplus (Hakuna Matata) 于 (Sun Apr 17 11:16:18 2011, 美东) 提到:
所以说呕吐是人体自我保护机制。
☆─────────────────────────────────────☆
rainingcats (喵喵) 于 (Sun Apr 17 11:22:06 2011, 美东) 提到:
为什么吐了以后立刻就好了呢?如果是因为毒素,那么毒素要进入中枢,才会产生头昏
等症状。而呕吐只是清除胃内容物,会避免更多毒素的影响,但不应该影响已经进入血
液的毒素代谢,为什么吐完1秒钟之内立刻缓解,这病理生理机制是什么?
☆─────────────────────────────────────☆
Aplusplus (Hakuna Matata) 于 (Sun Apr 17 11:46:40 2011, 美东) 提到:
我来试试回答你的问题:
1. 清除胃内容物是一部分, 减缓诱因。
2. 更重要的是:呕吐是vagal nerve response。When parasympathetic nerve
overactivated, sympathetic response will kick in. Your body will be under
adrenaline influence. 你的身体处于应激状态, 所以症状会得到暂时的缓解。
当然应激反应很复杂, 就不多说了。
☆─────────────────────────────────────☆
rainingcats (喵喵) 于 (Sun Apr 17 11:59:16 2011, 美东) 提到:
可能是这个原因吧
under
☆─────────────────────────────────────☆
Aplusplus (Hakuna Matata) 于 (Sun Apr 17 22:28:29 2011, 美东) 提到:
40 yrs old WF, presented with bilateral photophobia. She had this type
problems for the past several years and now her symptoms were getting
progressively worse. Her eye exams were reported normal. She had to wear
sunglasses all the time.
She told me at clinic that she had to go early since her daughter has an
endoscopy scheduled for mysterious abdominal pain.
PMH: Fibromyalgia, bipolar disorder, IBS. She also has chronic abdominal
pain and been scoped multiple times.
Social: Smokes 1 ppd; Social drink. No drug abuse.
FMH: Fibromyalgia, IBS.
ROS; Not remarkable except mentioned above.
PE: Vital: Normal.
All other PEs are not remarkable except severe photophobia, overactive BS
and mild diffuse abdominal tenderness.
Dx: Photophobia, unknown etiolgy
Your next step?
☆─────────────────────────────────────☆
skyscorpio (天之蝎子) 于 (Sun Apr 17 23:46:49 2011, 美东) 提到:
urine analysis, looking for something specific
☆─────────────────────────────────────☆
Aplusplus (Hakuna Matata) 于 (Mon Apr 18 08:38:38 2011, 美东) 提到:
Good for you!
You finally knocked the door. lol.
Would you share your google results with us?:}
☆─────────────────────────────────────☆
skyscorpio (天之蝎子) 于 (Mon Apr 18 21:06:05 2011, 美东) 提到:
porphyria caused by uroporphyrinogen decarboxylase deficiency, urine test to
look at uroporphyrinogen level.
☆─────────────────────────────────────☆
skyscorpio (天之蝎子) 于 (Mon Apr 18 21:18:54 2011, 美东) 提到:
包子包子!
LOL
to
☆─────────────────────────────────────☆
Aplusplus (Hakuna Matata) 于 (Mon Apr 18 21:34:40 2011, 美东) 提到:
The history is important. With a family history of IBS, chronic pain, mood
disorder and photosensitivity, please consider porphyria as a DDx.
There are different types of porphyria, and a blood test sometimes gives you
more info. I remember I ordered a bllood test then for the diagnosis.
Porphyria is very under-diagnosed as it can imitate a lot of other
complaints.
Typically, patients have been put through many expensive tests or procedures
before it is thought to check for porphyria. The cost of testing for
porphyria is not high compared to many other tests that are routinely done.
Controversially, the genetic predisposition for porphyria could be as high
as 1 in 500 in the population although active porphyria may be as low as 1
in 10,000.
Porphyria is incurable as yet. There is a high emphasis on prevention
strategies to keep it latent or symptom-free for a long life. But some
porphyrias can become serious progressive degenerative disorders if repeated
attacks are not stopped. Some treatments are available. like beta blockers.
Porphyria has been mistaken for: Guillain Barre, growing pains, Lupus,
eczema, somatisation disorders, epilepsy, MS, dermatology, depression,
appendicitis,Parkinson's and many, many more.
Suspicion is higher if symptoms come and go or appear after medication or
anaesthetic
to
☆─────────────────────────────────────☆
Aplusplus (Hakuna Matata) 于 (Mon Apr 18 21:39:16 2011, 美东) 提到:
The BZs are sponsored by skyscorpio and snowfox, many thanks to them.
☆─────────────────────────────────────☆
Aplusplus (Hakuna Matata) 于 (Sat Apr 23 00:25:49 2011, 美东) 提到:
Just read the case on the web, interesting.
"A woman came to the ER with severe abd pain, had dozens of visits through
the years, multiple specialists and told she had irritable bowel,
colonoscopies and CT in past c/w bowel edema, non-specific at times.
Laparoscopies also negative. Told she was a seeker on many occasions. I saw
her writhing in pain, typical narcs, etc. Repeat CT in ER with mod bowel
edema, also noticed she had a rash looked like hives. I asked, "do you break
out like this all the time these episodes happen."
"I think so." she said. "But they said it was always from the dilaudid or
morphine. and give me benadryl"
I gave her IV FFP in the ER and she completely resolved her rash and pain,
and she said..."OH my GOD, that stuff you gave me worked."
Labs showed severe C1 esterase deficiency. "
Have a nice weekend.
☆─────────────────────────────────────☆
snowfox01 (雪山飞狐) 于 (Sun Apr 24 12:52:46 2011, 美东) 提到:
Thanks for another excellent case.
saw
break
☆─────────────────────────────────────☆
Lexian (蒙古大夫) 于 (Sun Apr 24 20:02:30 2011, 美东) 提到:
This was in my mind yesterday, would actually be a good ethnic question.
OP: http://www.mitbbs.com/article_t0/MedicalCareer/31417961.html
What would you do if the OB doc hasn't made it here? baby's umbilical cord
came out from the birth canal. fetal heart rate shows decel. OB was called
but hasn't got here. You are the only doc available but it's not your
specialty. Do you cut or not?
☆─────────────────────────────────────☆
sunshadow (kageko) 于 (Sun Apr 24 20:22:13 2011, 美东) 提到:
呵呵,我压力大的时候,工作要紧的时候无论时间多久都不能吃东西。每吃必吐。
那种时候只喝水或者果汁。
☆─────────────────────────────────────☆
Aplusplus (Hakuna Matata) 于 (Sun Apr 24 21:34:41 2011, 美东) 提到:
Do not do it.
You could be a hero but you could face big troubles as well.
1. You might have a consent signed by you for anesthesia but not delievering
related procedures.
2. You are not licensed to deliever babys.
It really depended on the outcome. Even you did right thing but bad luck,
you would get into troubles.
Any OB doc here?
☆─────────────────────────────────────☆
Aplusplus (Hakuna Matata) 于 (Sun Apr 24 21:45:52 2011, 美东) 提到:
Your case is different. You are sick due to autonomous dysfunction (high
parasympathetic activity). This is consequence of stress which is adrenaline
related.
So in your case, the compensation for stress gives you the trouble.
What you need to do is:
1. Reduce the stress, of course, easy to say, difficult to do. I believe better sleep is a good way to relieve stress. Take something like melatonin if you do have insomnia.
2. Antiemetics should work. I like promethazine 25 mg po tid prn. and you can enjoy your meal under the stress.
☆─────────────────────────────────────☆
sunshadow (kageko) 于 (Sun Apr 24 21:55:50 2011, 美东) 提到:
啊,居然得到回复了。
那我多说两句,关于吐这个问题,我貌似是易吐体质(绝对不是减肥闹的),这几年才
开始有,压力大的时候吃了会吐,有时候来月经也会痛到吐,情绪激动也有两回,酒吐
那更是常事,汗。
为这个事情被周围的八卦男女怀疑有喜无数次。jiong。
adrenaline
can enjoy your meal under the stress.
☆─────────────────────────────────────☆
snowfox01 (雪山飞狐) 于 (Sun Apr 24 22:07:07 2011, 美东) 提到:
Thanks for the explanation. It was very educational.
delievering related procedures.
you would get into troubles.
☆─────────────────────────────────────☆
Aplusplus (Hakuna Matata) 于 (Sun Apr 24 22:43:50 2011, 美东) 提到:
Your case reminds me the sentance:
No good deed goes unpunished.
Do you know what it means? LOL!
This is the case of tonight, anyone jumps in? BZ available.
☆─────────────────────────────────────☆
Lexian (蒙古大夫) 于 (Sun Apr 24 22:51:14 2011, 美东) 提到:
I'd like to hear what these exam takers say...
☆─────────────────────────────────────☆
Aplusplus (Hakuna Matata) 于 (Sun Apr 24 22:52:15 2011, 美东) 提到:
C'mom, anybody can try except DrNewbie.
☆─────────────────────────────────────☆
Aplusplus (Hakuna Matata) 于 (Sun Apr 24 23:11:27 2011, 美东) 提到:
国内称植物神经忞乱, 不过最好让GI先看一下。
☆─────────────────────────────────────☆
Aplusplus (Hakuna Matata) 于 (Sun Apr 24 23:16:57 2011, 美东) 提到:
OK, I am not going to wait.
Dr.Lexian想做好事, 但很可能因此被sue。 这就叫好心不得好报, 反而被punished。
此所谓 No good deed goes unpunished。
☆─────────────────────────────────────☆
Lexian (蒙古大夫) 于 (Sun Apr 24 23:22:46 2011, 美东) 提到:
punished。
no, i'm afraid "afraid of getting sued" is not the right answer. I'd asked
the hospital risk management attorney about similar situation before. I'll
tell you what lawyer said.
☆─────────────────────────────────────☆
skyscorpio (天之蝎子) 于 (Sun Apr 24 23:29:01 2011, 美东) 提到:
因为不是您的病人,您可以选择做,也可以选择不做。如果您有自信做当然做了。而且,只要是在一个医生应该做的范围内,您的行为不用承担法律责任。
包子拿来!:)
☆─────────────────────────────────────☆
Lexian (蒙古大夫) 于 (Sun Apr 24 23:40:24 2011, 美东) 提到:
承担法律责任。
wrong answer.
☆─────────────────────────────────────☆
skyscorpio (天之蝎子) 于 (Sun Apr 24 23:42:17 2011, 美东) 提到:
因为不是您的病人,您可以选择做,也可以选择不做。如果您做了,而且,只要是在一
个医生应该做的范围内,您的行为不用承担法律责任。
包子拿来!:)
☆─────────────────────────────────────☆
Lexian (蒙古大夫) 于 (Sun Apr 24 23:51:14 2011, 美东) 提到:
do it, or don't, only one choice. and the reason why.
your answer is not correct, my dear BZ.
☆─────────────────────────────────────☆
Aplusplus (Hakuna Matata) 于 (Sun Apr 24 23:51:36 2011, 美东) 提到:
In case of emergency, you may opt to do whatever is needed to save the
infant. The question is how you justify such a procedure. If she is
unconscious, you are ok. But if she is conscious, oral consent is needed.
asked
☆─────────────────────────────────────☆
Lexian (蒙古大夫) 于 (Sun Apr 24 23:54:39 2011, 美东) 提到:
it's extreme emergency. she is crying, and begging "help me".
☆─────────────────────────────────────☆
skyscorpio (天之蝎子) 于 (Sun Apr 24 23:57:40 2011, 美东) 提到:
您老这个问题绝对超出我们的考试范围了,我拒绝羞愧!
☆─────────────────────────────────────☆
skyscorpio (天之蝎子) 于 (Mon Apr 25 00:07:31 2011, 美东) 提到:
不过我的行为学确实比较差,呼唤今年申请的真正的大牛出来
☆─────────────────────────────────────☆
Aplusplus (Hakuna Matata) 于 (Mon Apr 25 00:12:33 2011, 美东) 提到:
In case of emergency, you do not need consent, have nurse as the witness.
But unless you are comfortable about the diagnosis and the procedures(I
think you may have some trainings on OB), I would not recommend you to do it.
This is not CPR everyone has the training.
☆─────────────────────────────────────☆
Lexian (蒙古大夫) 于 (Mon Apr 25 00:13:11 2011, 美东) 提到:
well, i know one resident was FIRED over this issue. it is hard to
understand so don't feel ashamed.
shouldn't do it. because you are putting the mother's life in danger. No
matter how confident you are, you are not trained in C/S, which may be a
simple procedure in a pair of trained hand. It carries considerable amount
of risk, even life threatening for a untrained person. Should never overlook
mother's life for the baby's.
One of my junior resident was fired because of this. When he was on call for
OB, an emergency happened, OB doc rushed patient to operating room, demand
him (a 2nd yr resident) to put the patient to sleep so a C/S can be done. He
called attending but attending hasn't got here. He went ahead put the
patient to sleep, and the baby was delivered fine. But the attending refused
to sign the chart. This is the lesson we all learned - putting a pregnant
women to sleep carries enormous risk. You can't put baby's interest ahead of
the mother's.
☆─────────────────────────────────────☆
Aplusplus (Hakuna Matata) 于 (Mon Apr 25 00:17:40 2011, 美东) 提到:
That is what i have said. lol.
overlook
for
demand
He
☆─────────────────────────────────────☆
Lexian (蒙古大夫) 于 (Mon Apr 25 00:21:03 2011, 美东) 提到:
yes, but the lawyers can make it sound totally different, right?
☆─────────────────────────────────────☆
Aplusplus (Hakuna Matata) 于 (Mon Apr 25 10:26:41 2011, 美东) 提到:
Mecical practice has been dictated by these morons, sadly.
☆─────────────────────────────────────☆
REDpersimmon (大尾巴睡猫) 于 (Mon Apr 25 10:56:53 2011, 美东) 提到:
the 2nd year resident was in such bad luck, although he might have saved the
baby's life.
This lesson tells us--risk a baby's life and wait for attendings.
i feel the law needs to be changed--what if the 2nd year anethes. resident
was well trained on this and felt confident to do it without risking the
mother's life? Isn't that the patient's autonomy rules everything if the
patient is competent and conscious? What if the mother is willing to risk
her own life in order to save the baby?
☆─────────────────────────────────────☆
Lexian (蒙古大夫) 于 (Mon Apr 25 11:13:43 2011, 美东) 提到:
the
a good question. That would require an advance medical directive legally
constructed ahead of time.
as to the resident, it's not you feel you are confident or not, a resident
has only limited scope to practice medicine under the attending's medical
direction. They are not at the position to judge someone (including
themselves) qualify or not.
☆─────────────────────────────────────☆
snowfox01 (雪山飞狐) 于 (Mon Apr 25 11:28:40 2011, 美东) 提到:
"One of my junior resident was fired because of this. When he was on call
for OB, an emergency happened, OB doc rushed patient to operating room,
demand him (a 2nd yr resident) to put the patient to sleep so a C/S can be
done. He called attending but attending hasn't got here."
I have a question.
what should the poor resident do? there was a emergency and the surgeon
already gave him the order. Can he just tell the OB that I need to wait for
my Attending? and if something happen during the short waiting period, who
will be blamed ?
☆─────────────────────────────────────☆
REDpersimmon (大尾巴睡猫) 于 (Mon Apr 25 11:50:26 2011, 美东) 提到:
Thanks for answering my questions. So let's clarify that---All attendings,
of OB/GYN or Anesthesiology or any specialty--do not order anything out of
scope to us poor residents---as we are not physicians, we are physicians in
training! I'll never try to help anyone in public in those situations during
my residency since we're excluded from good samaritan law as we are not
Physicians! Is that right?
☆─────────────────────────────────────☆
REDpersimmon (大尾巴睡猫) 于 (Mon Apr 25 11:53:54 2011, 美东) 提到:
I have the same question. The resident seemed to be dead anyway.
It's not fair to him especially when the outcome is good. No one is sued,
why punish the resident for such a dilemma? Just warning is enough.
If the baby was dead by the time the attending showed up, I'm sure the
resident was the one to blame, at least by the OB
for
☆─────────────────────────────────────☆
Lexian (蒙古大夫) 于 (Mon Apr 25 14:14:10 2011, 美东) 提到:
He should try to get the approval from the attending (over the phone) first.
If the attending tell him to wait, he should wait. If the attending tell
him to go ahead, then go ahead. The attending is ultimately responsible for
the consequences. If you proceed without prior approval, and then you will
be liable.
for
☆─────────────────────────────────────☆
sunshadow (kageko) 于 (Mon Apr 25 17:01:13 2011, 美东) 提到:
But can Good Samaritan Law be applied here?
first.
for
☆─────────────────────────────────────☆
sunshadow (kageko) 于 (Mon Apr 25 17:04:40 2011, 美东) 提到:
好。谢谢。
☆─────────────────────────────────────☆
snowfox01 (雪山飞狐) 于 (Mon Apr 25 17:05:55 2011, 美东) 提到:
多谢,前辈的教育帖。
first.
for
☆─────────────────────────────────────☆
Lexian (蒙古大夫) 于 (Mon Apr 25 19:24:30 2011, 美东) 提到:
Good Samaritan Law doesn't apply here, you should not endanger the mother
while trying to help the baby, that is the different.
☆─────────────────────────────────────☆
yuzaiusa (Expecto Patronum!) 于 (Tue Apr 26 21:49:59 2011, 美东) 提到:
Good case. Thank u. BTW, are u a neurologist?
40 yrs old WF, presented with bilateral photophobia.
★ Sent from iPhone App: iReader Mitbbs 6.88 - iPad Lite
☆─────────────────────────────────────☆
daisyy (Daisy) 于 (Tue Apr 26 22:06:19 2011, 美东) 提到:
The resident get paid for the activities he performed in the hospital,
Good Samaritan Law can't be applied.
☆─────────────────────────────────────☆
yuzaiusa (Expecto Patronum!) 于 (Tue Apr 26 22:07:04 2011, 美东) 提到:
Here is something I always tell my interns: as a resident, we are
practicing under attending's license, maybe you don't agree with him/her,
you can discuss with them, but you should follow their decisions.
Thanks for answering my questions. So let's clarify that---All attendings,
of OB/GYN or ........
★ Sent from iPhone App: iReader Mitbbs 6.88 - iPad Lite
☆─────────────────────────────────────☆
sunshadow (kageko) 于 (Wed Apr 27 01:11:56 2011, 美东) 提到:
I wasnt talking about that intern. I was talking about Dr. Lexian's
imaginary case.
OB didn't show up, you were there for anesthesia. Pregger asked for your
help.
Can u do it w/o getting punished? Can GSL be applied here?
I guess I should ask my JD friends.
☆─────────────────────────────────────☆
REDpersimmon (大尾巴睡猫) 于 (Wed Apr 27 07:11:44 2011, 美东) 提到:
I did not mean to apply good samaritan law in this particular situation. I
meant in public, as a resident, we should not offer help because we are non-
physicians, and the law only protects real physicians.
☆─────────────────────────────────────☆
REDpersimmon (大尾巴睡猫) 于 (Wed Apr 27 07:13:31 2011, 美东) 提到:
This makes sense. Thanks!
We have a lot to learn about when to 请示。
first.
for
☆─────────────────────────────────────☆
Aplusplus (Hakuna Matata) 于 (Wed Apr 27 21:46:07 2011, 美东) 提到:
Today I got a phone call from a collegue. His previous nurse who is 54
years old female, presented with right foot pain for 4 days. The pain was
so severe that she could not walk. He did a MRI of L-spine and vascular u/s
which were all normal. ESR is 30. He wanted me take a 2nd look if he missed
something.
PMH. Lung Ca for 2 yrs. DM. HTN. Depression.
SH/FH/ROS: not remarkable.
What are you going to do at Exam if you suspect something?
☆─────────────────────────────────────☆
daisyy (Daisy) 于 (Wed Apr 27 22:08:53 2011, 美东) 提到:
In the hospital setting, I think it's hard to apply Good Samaritan Law
because lots of qualified professionals are around.
Try to ask other qualified doctors to help.
☆─────────────────────────────────────☆
sunshadow (kageko) 于 (Wed Apr 27 22:37:31 2011, 美东) 提到:
It makes sense, but still...
Curious to see the real cases.
Will get back if I find some n share w u
☆─────────────────────────────────────☆
daisyy (Daisy) 于 (Wed Apr 27 23:10:14 2011, 美东) 提到:
That will be very interesting.
☆─────────────────────────────────────☆
Aplusplus (Hakuna Matata) 于 (Wed Apr 27 23:47:21 2011, 美东) 提到:
Real cases?
For visitors, send to ER, maybe CPR if pulseless.
For patients, call code。
For Dr.lexien's case, do not do it. Call for help.
☆─────────────────────────────────────☆
dojo (麦地里的豆角-MS0) 于 (Thu Apr 28 00:19:18 2011, 美东) 提到:
Surprised no one gave a try yet.
DM -> peri neuropathy -> neuro exam of R foot, e.g. two-point discrim.
/s
missed
☆─────────────────────────────────────☆
Aplusplus (Hakuna Matata) 于 (Thu Apr 28 08:41:35 2011, 美东) 提到:
Good try.
Her right foot was so touch sensitive that a light touch triggered severe
pain. (allodynia) But her left foot was entirely normal.
The apperance of the foot looked same as the other foot.
☆─────────────────────────────────────☆
Aplusplus (Hakuna Matata) 于 (Thu Apr 28 17:18:15 2011, 美东) 提到:
Still no more people to try?
How about this picture?
/s
missed
☆─────────────────────────────────────☆
snowfox01 (雪山飞狐) 于 (Thu Apr 28 18:35:18 2011, 美东) 提到:
1.我的第一印象是:肺腺癌导致高钙血症---骨折(2年正好是该出现骨破坏).
2.第二印象是:小细胞癌化疗导致的副作用---神经痛。(小细胞癌活到2年?不容易)
3.看了你的图片----猜是癌症导致的免疫低下,引起的带状疱疹感染。
☆─────────────────────────────────────☆
Aplusplus (Hakuna Matata) 于 (Thu Apr 28 19:21:34 2011, 美东) 提到:
The first thing I did was to do a thorough skin exam. I found the rash on
the right buttock just like the pic. So the diagnosis was herpes zoster
radiculopathy.
Anyone got the shinges before? I got once at age 22, it was so painful as
hell!
Her oncologist saw her on same day and called me: " I have been seeing this
all the time".
☆─────────────────────────────────────☆
Aplusplus (Hakuna Matata) 于 (Mon May 2 23:18:22 2011, 美东) 提到:
I did one month OB prior to internal medicine. I remembered there was a
patient with FOU. After the rounds, the attending brought me back to the
patient's room. He said "I'm just afraid I missed something." Then he put
on the gloves and did a thorough skin exam including all the skin folds and
grooves. You know what? He did find a small decubitus ulcer at the bottom
of right buttock. I was not following that case but I really admired his
attitude.
I wish I could find the problem by myself. Since then, I have been so
careful on PEs and one time an ICU attending commented on my PE as
meticulous. I was told by my chairman afterwards.
☆─────────────────────────────────────☆
aoisora (简单就好) 于 (Tue May 3 18:45:17 2011, 美东) 提到:
what is fuo?
☆─────────────────────────────────────☆
aoisora (简单就好) 于 (Tue May 3 18:46:43 2011, 美东) 提到:
sorry about the typo, I meant fou in your posting. thanks.
☆─────────────────────────────────────☆
Aplusplus (Hakuna Matata) 于 (Tue May 3 19:51:52 2011, 美东) 提到:
Fever of Unknown Origin
s********o
发帖数: 3319
22
这是A++前辈在麦地最有影响的一个帖子,可以说是他在买卖提挖到的第一桶金。现在转给新版, 做个永久纪念。版二可以考虑收入精华区.
再次向A++前辈致谢!
A*******s
发帖数: 9638
23
你喜欢吃蝎子的那张照片吗?
谢谢蝎子版主。

【在 s********o 的大作中提到】
: 这是A++前辈在麦地最有影响的一个帖子,可以说是他在买卖提挖到的第一桶金。现在转给新版, 做个永久纪念。版二可以考虑收入精华区.
: 再次向A++前辈致谢!

s********o
发帖数: 3319
24
呵呵,我还记得我刚上台时您开口就叫我“毒蝎”。咱们都应该庆幸,我当时忍住了没有骂还,否则江湖中就会少了一个门派,多了一段恩怨和许多冤魂,LOL
我要淡出江湖一阵子了。您老好好干。如果觉得麦地版有什么帖子你们比较感兴趣可以自己转过来讨论,像很多求医问药的帖子等等。

【在 A*******s 的大作中提到】
: 你喜欢吃蝎子的那张照片吗?
: 谢谢蝎子版主。

A*******s
发帖数: 9638
25
No sweat, LOL!

没有骂还,否则江湖中就会少了一个门派,多了一段恩怨和许多冤魂,LOL
以自己转过来讨论,像很多求医问药的帖子等等。

【在 s********o 的大作中提到】
: 呵呵,我还记得我刚上台时您开口就叫我“毒蝎”。咱们都应该庆幸,我当时忍住了没有骂还,否则江湖中就会少了一个门派,多了一段恩怨和许多冤魂,LOL
: 我要淡出江湖一阵子了。您老好好干。如果觉得麦地版有什么帖子你们比较感兴趣可以自己转过来讨论,像很多求医问药的帖子等等。

A*******s
发帖数: 9638
26
今天下午看到一个case, 与大家分享:
67 yrs old WF with a history of A-fib, presented to ER with diaphoresis and
fatigue for 2 days. She has been on Pradaxa for a-fib. Cardiology suspected CHF exacerbation.
PMH: CAD, CHF and a-fib. No DM.
Social/FMH/ROS: Not remarkable.
On exam, Vital was stable. HR was only 76.
She was noticed to have an anisocoria. She blamed her cataract surgery 10 years ago for the cause of unequal sized pupils. But her family believed it was new onet.
Pupil OS 7 mm, Sluggish to light. OD 3 mm, reactive. ocular movement was
normal. Had wet skins and no reflexes.
All other PEs were not remarkable. Lung was clear. No peripheral edema.
Brain MRI was normal.
What did she have?
See the attached photo below.



在转给新版, 做个永久纪念。版二可以考虑收入精华区.

【在 s********o 的大作中提到】
: 这是A++前辈在麦地最有影响的一个帖子,可以说是他在买卖提挖到的第一桶金。现在转给新版, 做个永久纪念。版二可以考虑收入精华区.
: 再次向A++前辈致谢!

V*****G
发帖数: 337
27
Adie Pupil? Or medication effect?

and
suspected CHF exacerbation.
years ago for the cause of unequal sized pupils. But her family believed it
was new onet.

【在 A*******s 的大作中提到】
: 今天下午看到一个case, 与大家分享:
: 67 yrs old WF with a history of A-fib, presented to ER with diaphoresis and
: fatigue for 2 days. She has been on Pradaxa for a-fib. Cardiology suspected CHF exacerbation.
: PMH: CAD, CHF and a-fib. No DM.
: Social/FMH/ROS: Not remarkable.
: On exam, Vital was stable. HR was only 76.
: She was noticed to have an anisocoria. She blamed her cataract surgery 10 years ago for the cause of unequal sized pupils. But her family believed it was new onet.
: Pupil OS 7 mm, Sluggish to light. OD 3 mm, reactive. ocular movement was
: normal. Had wet skins and no reflexes.
: All other PEs were not remarkable. Lung was clear. No peripheral edema.

A*******s
发帖数: 9638
28
Congratulations!
Adie's syndrome.

it

【在 V*****G 的大作中提到】
: Adie Pupil? Or medication effect?
:
: and
: suspected CHF exacerbation.
: years ago for the cause of unequal sized pupils. But her family believed it
: was new onet.

V*****G
发帖数: 337
29
Thanks for your cases, always can learn more!!!
For that case of food poisoning, I doubt it is staphy aureus, because the
main part of presentation is vertigo. The only thing I find close to that
presentation is Botulism. (I didn't reply at that time is because I cannot
surf often), I really think answer should be Clostridium Botulinum
http://www.livestrong.com/article/111800-botulism-poisoning-sym
Vision Changes
Patients who develop botulism poisoning typically experience vision changes
within 12 to 72 hours after exposure to the contaminated food, explain Food
Safety, a website managed by the U.S. Department of Health and Human
Services. Initially, botulism patients can develop vertigo, which causes
them to feel as though their surroundings are spinning uncontrollably.
Blurred, double or cloudy vision can also arise, making it difficult for
infected patients to see normally. These vision symptoms can contribute to
additional side effects, including dizziness and headache.
Read more: http://www.livestrong.com/article/111800-botulism-poisoning-symptoms/#ixzz1OGlQ71bZ

【在 A*******s 的大作中提到】
: Congratulations!
: Adie's syndrome.
:
: it

V*****G
发帖数: 337
30
I have a real case, just roughly know the history, please bear with me.
A 62 yo w m c/o left shoulder pain for several days
Pain is dull, 6/10,
Caught a cold several days before onset of shoulder pain
With fatigue, slight short of breath
Non smoker
Past history is HTN x 15 years, on diuretics
No travel history
No trauma/injury history
Occupation: office based job
PE: normal
What is the next step?
相关主题
【Intern日记】Cardiothoracic Surgery【Case discussion】 Vision loss
版上有CT surgeon么Don't jump, please
An interesting case to share病例有奖竞猜, Ageusia and anosmia
进入Medicalpractice版参与讨论
A*******s
发帖数: 9638
31
X-rays of the chest and shoulder.
Dissection?

【在 V*****G 的大作中提到】
: I have a real case, just roughly know the history, please bear with me.
: A 62 yo w m c/o left shoulder pain for several days
: Pain is dull, 6/10,
: Caught a cold several days before onset of shoulder pain
: With fatigue, slight short of breath
: Non smoker
: Past history is HTN x 15 years, on diuretics
: No travel history
: No trauma/injury history
: Occupation: office based job

d****y
发帖数: 2180
32
The pain should be sharp in dissection.

【在 A*******s 的大作中提到】
: X-rays of the chest and shoulder.
: Dissection?

A*******s
发帖数: 9638
33
Typical case, yes。
Not sure if the pain is associated with breathing.

【在 d****y 的大作中提到】
: The pain should be sharp in dissection.
s*******1
发帖数: 428
34
bacterial endocarditis?
Next step: blood culture, echo.

【在 V*****G 的大作中提到】
: I have a real case, just roughly know the history, please bear with me.
: A 62 yo w m c/o left shoulder pain for several days
: Pain is dull, 6/10,
: Caught a cold several days before onset of shoulder pain
: With fatigue, slight short of breath
: Non smoker
: Past history is HTN x 15 years, on diuretics
: No travel history
: No trauma/injury history
: Occupation: office based job

V*****G
发帖数: 337
35
this guy finally went to the hospital. It turns out this guy had a heart
attack, 3 vessel stenosis, had a CABG
before released from hospital.

【在 V*****G 的大作中提到】
: I have a real case, just roughly know the history, please bear with me.
: A 62 yo w m c/o left shoulder pain for several days
: Pain is dull, 6/10,
: Caught a cold several days before onset of shoulder pain
: With fatigue, slight short of breath
: Non smoker
: Past history is HTN x 15 years, on diuretics
: No travel history
: No trauma/injury history
: Occupation: office based job

A*******s
发帖数: 9638
36
I was looking for zebras.
Yes, always check cardiac enzymes and EKG for the referred pain.

【在 V*****G 的大作中提到】
: this guy finally went to the hospital. It turns out this guy had a heart
: attack, 3 vessel stenosis, had a CABG
: before released from hospital.

A*******s
发帖数: 9638
37
昨天有同仁对我的一个case感兴趣, 我今天查了下病历, 给大家写个完整的。
53 yrs old WM, previously healthy, presented with intractable fatigue and
weight loss for 3 months. A comprehensive workup by his PCP including a CT
of chest was reported as unremarkable.
PMH: Asthma, seeing a pulmonologist who reviewed chest CT and agreed with
radiologist's report.
Soical. Smoker 1 ppd. No ETOH/Drug.
FMH: neg for CA.
ROS: Not remarkable, no diplopia, dysphagia, no fever, no joint pain, no
muscle ache.
PE: Vital normal.
No positive findings on PE.
Lab: CBC/CMP: nl, ESR/CRP: nl; CPK: nl.
CT of Chest: single lymphadenopathy, non-specific.
I have no clue at the first visit. I ordered a ACHR AB to r/o Myasthenia
Gravis. and ampirically started him on Mestinon. 1 week later, he returned
and told me he had improved after being on mestinon. But his ACHR was neg,
as well as MUSK antibody. I started to cast doubt on the diagnosis. So I re
-examed him and again everything was normal except mild weakness on right
hand grip, very, very subtle. So I went ahead for brain CT. Why CT? Because it
was cheap and no preapproval, and I was not that sure.
Surprising there was a tumor in the left frontal lobe. I suddenly understood
what was going on. I called the radiologist who read the chest CT and requested
him to do the lymph node biopsy. The pathology report comfirmed small call
CA. I sent him to a famous academic cancer center.
Interestingly, I was not able to taper off the mestinon. Anti-Hu antibody
title was very high. I started to suspect he has ELS instead of MG.
The academic cancer center sent him to neurology department and they
comfirmed he has ELS, one of the paraneoplastic syndrome. They recommended
him to continue mestinon for ELS, interestingly.

It has been 2.5 yrs already. He has been under serial chemo/radiation
treatements and can not have regular clinic visits due to immunosuppression.
Conclusion:
This patient's fatigue symptom is not from brain metastasis or lung Ca per
se. Instead, ELS caused his clinic symptoms and led to the earlier diagnosis.
V*****G
发帖数: 337
38
Nice, first time to know how ELS presented in the real case. It is just one
thing, that's it, how nice it is! And again, PE is very important and assume
everything is real (not the opposite).

CT

【在 A*******s 的大作中提到】
: 昨天有同仁对我的一个case感兴趣, 我今天查了下病历, 给大家写个完整的。
: 53 yrs old WM, previously healthy, presented with intractable fatigue and
: weight loss for 3 months. A comprehensive workup by his PCP including a CT
: of chest was reported as unremarkable.
: PMH: Asthma, seeing a pulmonologist who reviewed chest CT and agreed with
: radiologist's report.
: Soical. Smoker 1 ppd. No ETOH/Drug.
: FMH: neg for CA.
: ROS: Not remarkable, no diplopia, dysphagia, no fever, no joint pain, no
: muscle ache.

d**o
发帖数: 618
39
Thank you very much for the detailed description. This patient is really
lucky!

【在 A*******s 的大作中提到】
: 昨天有同仁对我的一个case感兴趣, 我今天查了下病历, 给大家写个完整的。
: 53 yrs old WM, previously healthy, presented with intractable fatigue and
: weight loss for 3 months. A comprehensive workup by his PCP including a CT
: of chest was reported as unremarkable.
: PMH: Asthma, seeing a pulmonologist who reviewed chest CT and agreed with
: radiologist's report.
: Soical. Smoker 1 ppd. No ETOH/Drug.
: FMH: neg for CA.
: ROS: Not remarkable, no diplopia, dysphagia, no fever, no joint pain, no
: muscle ache.

A*******s
发帖数: 9638
40
I felt I was flattered. :)

【在 d**o 的大作中提到】
: Thank you very much for the detailed description. This patient is really
: lucky!

相关主题
病例有奖竞猜, Ageusia and anosmia【病例讨论】Anisocoria
Case #2 Fever in Traveler【有奖征贴】说一说你亲眼见过的最疑难病例
肥胖是一种疾病大家说说, 以你自己的理解, 医学是什么东东?
进入Medicalpractice版参与讨论
f******w
发帖数: 10267
41
蝎子好心转过来的,大家复习复习也能学东西。
1 (共1页)
进入Medicalpractice版参与讨论
相关主题
求教:妈妈冠心病的治疗选择(附CT图)Case #2 Fever in Traveler
冠心病,CT、PCI、CABG......砍掉Cable贴!!肥胖是一种疾病
【Intern日记】Cardiothoracic Surgery【病例讨论】Anisocoria
版上有CT surgeon么【有奖征贴】说一说你亲眼见过的最疑难病例
An interesting case to share大家说说, 以你自己的理解, 医学是什么东东?
【Case discussion】 Vision lossHeparin drip
Don't jump, please新医生上路最常犯的9个错误
病例有奖竞猜, Ageusia and anosmiaFunny story about OSH
相关话题的讨论汇总
话题: ct话题: els话题: pain话题: pe话题: mestinon